Μαθηματικά Προβλήματα

Βιβλίο για την επίλυση προβλημάτων Συνδυαστικής
για περισσότερες πληροφορίες πατήστε στην κόκκινη γραμμή

Συνδυαστική Απαρίθμηση και Συνδυαστική Γεωμετρία



__________________________________________________________
Για το πρόβλημα του εξισωτή τριγώνου
Με αφορμή μία εργασία που δημοσιεύτηκε στο περιοδικό «Ευκλείδης Γ΄»
Ανδρέας Πούλος
       Οι συνάδελφοι μαθηματικοί Δημήτρης Ντρίζος και Γιώργος Ρίζος δημοσίευσαν πρόσφατα στο περιοδικό «Ευκλείδης Γ΄», τεύχος 81, Β΄ εξάμηνο 2014, μία εργασία τους έκτασης 20 σελίδων με τίτλο «Το πρόβλημα του εξισωτή τριγώνου: πλήρης μελέτη για κάθε είδος τριγώνου». Επειδή θεωρώ το θέμα πολύ ενδιαφέρον, όχι μόνον από προσωπική άποψη, αλλά και από συζητήσεις με άλλους συναδέλφους, αισθάνομαι την υποχρέωση να ενημερωθεί ένα ευρύτερο κοινό μαθηματικών, οι οποίοι δεν διαβάζουν τον «Ευκλείδη Γ΄», αλλά θα έβρισκαν χρήσιμο για τη δουλειά τους την αντιμετώπιση αυτού του θέματος. Θα παρουσιάσουμε αυτή την εργασία, τον τρόπο με τον οποίο οι δύο συγγραφείς εργάστηκαν και τις τεχνικές που ανέπτυξαν για να αντιμετωπίσουν το συγκεκριμένο πρόβλημα. Επίσης, θα αναφερθούμε για την αξία της στη μαθηματική μας βιβλιογραφία.
       Κατ’ αρχάς, είναι αναγκαίο να σημειώσουμε ότι ως εξισωτή, οι συγγραφείς ορίζουν μία ευθεία γραμμή, η οποία χωρίζει ένα επίπεδο χωρίο (στην προκειμένη περίπτωση ένα τριγωνικό χωρίο) σε δύο σχήματα ισοδύναμα και ισοπεριμετρικά. Οι συγγραφείς στην περίληψη της δημοσίευσής τους, ουσιαστικά στο εισαγωγικό σημείωμα για το πρόβλημα, τονίζουν ότι έχουν λάβει υπόψη τους τη βιβλιογραφία στην ελληνική και στην αγγλική γλώσσα, που αναφέρεται στο πρόβλημα της εύρεσης του εξισωτή τριγώνου. Θεωρούν ως πλεονέκτημα τον τρόπο προσέγγισης του προβλήματος, με εξαιρετικά λίγες απαιτήσεις σε μαθηματικές γνώσεις, επιπέδου Λυκείου. Αυτός ήταν και ένας από τους λόγους που επιχείρησαν να την δημοσιοποιήσουν σε μαθηματικό περιοδικό.
       Το ότι το πρόβλημα είναι αρκετά δύσκολο και απαιτητικό προκύπτει από το δεδομένο ότι το χρησιμοποιούσαν ως κριτήριο επιλογής για τα σχολεία ταλαντούχων στα Μαθηματικά της πρώην Σ.Ε., βλέπε το άρθρο του A. Shen, (Shen, 1994).
      Οι συγγραφείς εργάστηκαν με τη μελέτη των ειδικών περιπτώσεων, ειδικών τριγώνων, ώστε να καταλήξουν στο γενικό συμπέρασμα, δηλαδή στην επίλυση του προβλήματος για κάθε είδος τριγώνου. Διέκριναν δύο βασικές περιπτώσεις:
α) ο εξισωτής να διέρχεται από μία κορυφή του δεδομένου τριγώνου – στην περίπτωση αυτή αποδεικνύουν ότι το πρόβλημα έχει λύση, μόνο αν ο εξισωτής διέρχεται από την κορυφή ισοσκελούς τριγώνου – και
β) ο εξισωτής τέμνει τις πλευρές του τριγώνου. Στην περίπτωση αυτή, το πρώτο τους βήμα ήταν να περιγράψουν τις αλγεβρικές σχέσεις που συνδέουν τα μήκη των πλευρών του τριγώνου και τις αποστάσεις των σημείων από τα οποία διέρχεται ο εξισωτής σε σχέση με τις κορυφές του τριγώνου. Ειδικά για το ισόπλευρο τρίγωνο, απέδειξαν ότι δεν υπάρχει εξισωτής που να τέμνει τις πλευρές του σε εσωτερικά σημεία.
      Σημειώνουμε ότι κατά τη μελέτη του προβλήματος για ισόπλευρο και για ισοσκελές τρίγωνο, το μαθηματικό εργαλείο που χρησιμοποίησαν είναι στοιχειώδης Άλγεβρα, οι ιδιότητες της διακρίνουσας μιας δευτεροβάθμιας πολυωνυμικής εξίσωσης και στοιχειώδεις ανισότητες.
     Στην περίπτωση του ισοσκελούς τριγώνου απέδειξαν ότι υπάρχουν δύο εξισωτές, που τέμνουν τις πλευρές του, οι οποίοι μάλιστα είναι συμμετρικοί ως προς τον άξονα συμμετρίας του σχήματος. Μάλιστα, το ενδιαφέρον για την περίπτωση του ισοσκελούς τριγώνου είναι και η ανακάλυψη μιας αναγκαίας συνθήκης για να υπάρχει εξισωτής. Κατέληξαν ότι η συνθήκη ύπαρξης εξισωτή για ισοσκελή τρίγωνα με πλευρές β = γ είναι η ,(1). Αυτή η συνθήκη με μία ισοδύναμη μορφή είχε παρουσιαστεί από τον Δ. Κοντοκώστα, (Kodokostas, 2010). Αυτός ανακάλυψε την ακριβή σημασία της συνθήκης για τη γωνία  σε σχέση με το πλήθος των λύσεων του προβλήματος. Η γωνία απέναντι από την βάση του ισοσκελούς τριγώνου πρέπει να έχει άνοιγμα από 48ο 56’ 23΄΄ έως 60ο. Δηλαδή, δεν υπάρχει εξισωτής για όλα τα ισοσκελή τρίγωνα, π.χ. για τα αμβλυγώνια ισοσκελή τρίγωνα.
       Τέλος, ένα επίσης ενδιαφέρον εύρημα για τη διερεύνηση του προβλήματος είναι ότι δεν υπάρχει εξισωτής, ο οποίος να τέμνει εσωτερικά τη βάση του και μία από τις δύο ίσες πλευρές του.

Η μελέτη της ύπαρξης εξισωτή για σκαληνό τρίγωνο είναι η δυσκολότερη και η πλέον ενδιαφέρουσα. Για να διευκολύνουμε την κατανόηση των αποτελεσμάτων της μελέτης, ας θεωρήσουμε ότι για τις πλευρές α, β, γ του ισοσκελούς τριγώνου ισχύει η διάταξη β > γ > α. Οι συγγραφείς αποδεικνύουν ότι:
1. Δεν υπάρχει εξισωτής που να τέμνει εσωτερικά τις δύο μικρότερες πλευρές του.
2. Υπάρχει πάντα, δηλαδή ανεξάρτητα από το μήκος των πλευρών, ένας εξισωτής, ο οποίος   τέμνει εσωτερικά τη μεγαλύτερη και τη μικρότερη πλευρά, την β και την α, αντίστοιχα.
3. Υπάρχουν δύο εξισωτές, οι οποίοι τέμνουν τη μεγαλύτερη και τη μεσαία (σε μήκος) πλευρά, αρκεί να ισχύει μία συνθήκη την οποία αποδεικνύουν. 
Η συνθήκη είναι η (α + β + γ)2  ≥ 8βγ, (2).
Στην περίπτωση που είναι , οι συγγραφείς αναφέρουν τη συνθήκη που περιγράφει ο Δημήτρης Κοντοκώστας σε σχετική με τον εξισωτή εργασία του στο περιοδικό Mathematical Magazine, (Kodokostas, 2010), ότι ο εξισωτής δηλαδή, διέρχεται από το έγκεντρο του τριγώνου και είναι κάθετος στη εσωτερική διχοτόμο την μεταξύ των πλευρών β και γ.

Τελικά, οι δύο συγγραφείς κατέληξαν στα ακόλουθα συμπεράσματα.
1.Σε κάθε ισόπλευρο τρίγωνο οι άξονες συμμετρίας του είναι εξισωτές του.
2. Σε κάθε ισοσκελές τρίγωνο,
α) ο άξονας συμμετρίας του είναι εξισωτής του,
β) υπάρχουν δύο εξισωτές, συμμετρικοί ως προς τον άξονα συμμετρίας του,  που τέμνουν τις ίσες πλευρές β, γ, αν και μόνο αν ισχύει η συνθήκη (1).
γ) Δεν υπάρχει εξισωτής που να τέμνει σε εσωτερικά τους σημεία τις πλευρές α, β, όπου α είναι η βάση του ισοσκελούς τριγώνου.
3. Σε κάθε σκαληνό τρίγωνο ΑΒΓ με β > γ > α,
α) δεν υπάρχει εξισωτής που να τέμνει σε εσωτερικά τους σημεία τις πλευρές α, γ.
β) υπάρχει πάντα ένας εξισωτής που τέμνει σε εσωτερικά τους σημεία τις πλευρές α, β.
γ) υπάρχουν δύο εξισωτές που τέμνουν τις πλευρές β, γ σε εσωτερικά τους σημεία, αν και μόνο αν ισχύει (α + β + γ)2  ≥ 8βγ.
          Εφόσον τα μήκη των τμημάτων κ, λ είναι ρίζες δευτεροβάθμιας εξίσωσης με πραγματικούς συντελεστές, τα τμήματα αυτά είναι κατασκευάσιμα σε κάθε περίπτωση, οπότε και οι εξισωτές οποιουδήποτε τριγώνου μπορούν να κατασκευαστούν. Σε κάθε περίπτωση, οι συγγραφείς σημειώνουν ότι οι εξισωτές εφόσον υπάρχουν, είναι κατασκευάσιμοι με κανόνα και διαβήτη.

       Μετά την συνοπτική περιγραφή της λύσης του προβλήματος που μας έδωσαν οι δύο συγγραφείς, θα επιχειρήσουμε μία αποτίμηση της εργασίας τους σε σχέση με τις παλαιότερες λύσεις που προτάθηκαν για αυτό.
       Ο George Berzsenyi (Berzsenyi,1997) στο συνοπτικό του άρθρο για τον εξισωτή τριγώνου είχε θέσει το ερώτημα-εικασία, αν υπάρχει τρίγωνο με περισσότερους από 3 εξισωτές. Συγκεκριμένα αναφέρει, «έχω την πεποίθηση ότι το άνω φράγμα του πλήθους των εξισωτών τριγώνου είναι τρία». Σε αυτό απάντησαν εμμέσως οι συγγραφείς με τα συμπεράσματά τους. Δεν υπάρχει τέτοιο τρίγωνο. Ο Berzsenyi στο ίδιο άρθρο ανέφερε την εικασία, την οποία είχε διατυπώσει ο καθηγητής H. Bailey ότι δεν υπάρχει τρίγωνο με ακριβώς δύο εξισωτές. Ο Δημήτρης Κοντοκώστας γνώριζε την εικασία και απάντησε σαφώς στην εργασία του (Kodokostas, 2010) ότι αυτή είναι λανθασμένη, δηλαδή απέδειξε ότι υπάρχουν τρίγωνα με ακριβώς δύο εξισωτές. Οι συγγραφείς, στο εύρημα 3γ απαντούν εμμέσως.
         Θεωρούμε χρήσιμο να αναφέρουμε, ότι στο ίδιο άρθρο ο G. Berzsenyi διατυπώνει την πρόκληση ότι είναι καλό να μελετηθεί το τρισδιάστατο ανάλογο του προβλήματος, δηλαδή η τομή τετραέδρου από επίπεδο, το οποίο να το χωρίζει σε δύο στερεά ίσου όγκου και ίσου επιφανειακού εμβαδού. Αυτό δεν σημαίνει ότι η εργασία των δύο συγγραφέων μειονεκτεί, επειδή δεν απαντά και στο πρόβλημα αυτό, εξάλλου κάθε εργασία έχει ένα συγκεκριμένο στόχο, δεν μπορεί να απαντά σε όλα τα ανοικτά ερωτήματα. Όπως παρατήρησα στην βιβλιογραφία για τον εξισωτή τριγώνου και οι υπόλοιποι ερευνητές δεν απάντησαν στην πρόκληση του Berzsenyi. Απλά την αναφέρω, επειδή συνδέεται άμεσα με το θέμα που εξετάζουμε.
         Αναφέραμε και προηγούμενα ότι οι συγγραφείς εργάστηκαν κυρίως με αλγεβρικά μέσα, δεν ενδιαφέρθηκαν για μία απόδειξη του θεωρήματος ότι «ο εξισωτής ενός τριγώνου διέρχεται αναγκαστικά από το έγκεντρο του», ίσως επειδή διαπίστωσαν ότι είχαν προηγηθεί άλλοι (Kodokostas 2010, Kung 2002, De et al, 2014). Υποθέτουμε ότι για τον ίδιο λόγο, οι συγγραφείς δεν ασχολήθηκαν και με την πρόταση «μία ευθεία που διέρχεται από το έγκεντρο ενός τριγώνου και το χωρίζει σε δύο ισεμβαδικά σχήματα, τότε το χωρίζει και σε δύο ισοπεριμετρικά σχήματα» ή με τη γενικότερη πρόταση «μία ευθεία που διέρχεται από το έγκεντρο ενός τριγώνου και το χωρίζει σε δύο σχήματα, τότε ο λόγος των εμβαδών των σχημάτων αυτών είναι ίσος με το λόγο των περιμέτρων τους».
          Η πρωτοτυπία της προσπάθειας των δύο συγγραφέων βρίσκεται κυρίως στα απλά αλγεβρικά εργαλεία που χρησιμοποίησαν για να προσεγγίσουν το πρόβλημα. Αντίστοιχα, ο Κοντοκώστας, (Kodokostas 2010) στην πρώτη φάση της έρευνάς του χρησιμοποίησε καθαρά γεωμετρικές τεχνικές και στη δεύτερη φάση Τριγωνομετρία και παραγώγους για να υπολογίσει τα ακρότατα σε ένα διάστημα μιας συνάρτησης, η οποία ήταν γνησίως φθίνουσα. Ας σημειωθεί ότι ο ίδιος έθεσε το ερώτημα αν η συνθήκη παίζει ρόλο και σε άλλα προβλήματα των Μαθηματικών, κάτι το οποίο ο ίδιος δεν γνώριζε. Βεβαίως, η εργασία του Δημήτρη Κοντοκώστα είναι υποδειγματική ως προς τον τρόπο που χειρίζεται το θέμα του, ως προς τα συμπεράσματα που εξάγει και ως προς τις τεχνικές που χρησιμοποιεί. Για τον λόγο αυτό άλλωστε είναι και σημείο αναφοράς στο σχετικό πρόβλημα.
      Σημειώνουμε ότι και ο Ross Honsberger, (Honsberger, 2004), βιβλία του οποίου  έχει εκδώσει η Μαθηματική Εταιρεία των Η.Π.Α. για να επιλύσει αυτό το πρόβλημα έκανε χρήση Διαφορικού Λογισμού.
        Οι συγγραφείς χειρίζονται το θέμα τους με έναν τρόπο που είναι κατανοητός και από μαθητές Λυκείου. Θεωρούμε ότι η μακρά θητεία τους στην εκπαίδευση έπαιξε ρόλο σε αυτή την προσπάθεια, να γίνει δηλαδή κατανοητό το συγκεκριμένο πρόβλημα και η αντιμετώπισή του από όσο το δυνατόν ευρύτερο κοινό ενδιαφερομένων.
        Επίσης, καλό είναι να τονίσουμε ότι οι δύο συγγραφείς μεριμνούν και για την ευκλείδεια κατασκευή των εξισωτών τριγώνου, κάτι που δεν συναντάμε σε άλλες σχετικές εργασίες. Προαναφέραμε, ότι επειδή τα μήκη των τμημάτων στα οποία ο εξισωτής τέμνει τις πλευρές του τριγώνου αποτελούν ρίζες δευτεροβάθμιας εξίσωσης με πραγματικούς συντελεστές, τα τμήματα αυτά είναι κατασκευάσιμα σε κάθε περίπτωση, συνεπώς οι εξισωτές μπορούν να κατασκευαστούν με κανόνα και διαβήτη.
        Εν τέλει θεωρούμε ότι έχει ενδιαφέρον να μελετήσουμε τον τρόπο με τον οποίο οι δύο συγγραφείς αντιμετώπισαν αυτό το απαιτητικό και ακανθώδες πρόβλημα μέσω της σχολικής Άλγεβρας και ότι αυτή τους η προσπάθεια αποτελεί παράδειγμα για αντίστοιχα προβλήματα. Προσωπικά, η εργασία αυτή θα μου χρησιμεύσει σε σεμινάρια επίλυση μαθηματικών προβλημάτων, τα οποία θα πραγματοποιηθούν στα Π.Ε.Κ. Θεσσαλονίκης.

ΒΙΒΛΙΟΓΡΑΦΙΚΕΣ ΑΝΑΦΟΡΕΣ
Berzsenyi George, (1997). Εξισωτής τριγώνου. Μία έξυπνη ευθεία που έχει δύο ρόλους. Περιοδικό Quantum. Εκδόσεις Κάτοπτρο, τόμος 4, τεύχος 3, σελ. 47.
Honsberger Ross, (2004). Mathematical Delighs. M.A.A. p. 71-74.
Kodokostas Dimitrios, (2010). Triangle Equalizers. Mathematics Magazine83: 2, 141-146.
Kung H. Sidney, (2002). A Line through the Incenter of a Triangle. Mathematics Magazine, 75, 3, 214. 
Ντρίζος Δημήτρης & Γιώργος Ρίζος, (2014). Το πρόβλημα του εξισωτή τριγώνου: πλήρης μελέτη για κάθε είδος τριγώνου. Περιοδικό Ευκλείδης Γ, τεύχος 81, σελίδες 1-20.
Shen, Α., (1994). Entrance Examinations to the Mekh-mat, The Mathematical Intelligencer, Vol. 16, No.4, 6–10.
Todd Anthony, (1999).  Bisecting a Triangle. Pi Mu Epsilon11: 1, 31-37.
Todd Anthony, (2011). Letter to the Editor, Mathematics Magazine84: 5, 396.
Διευθύνσεις από το Διαδίκτυο που αναφέρονται στο θέμα:



Προβλήματα που έχω αναρτήσει
στο Φόρουμ www.mathematica.gr


1. Σάββατο 15 Ιανουαρίου 2011
Με αφορμή το 3ο θέμα του διαγωνισμού Ευκλείδης 2011 της ΕΜΕ για την Α Λυκείου,
θέτω για απόδειξη το παρακάτω κριτήριο.
Ένα παραλληλόγραμμο, στο οποίο μία διαγώνιός του είναι διχοτόμος μιας γωνίας του, είναι ρόμβος.

2. Δευτέρα 31 Ιανουαρίου 2011
Δίνεται ένα επίπεδο Ρ και ένα σημείο του έστω Α.
Αν Β ένα σημείο εκτός του επιπέδου Ρ, να βρεθούν τα σημεία Χ του Ρ για τα οποία ο λόγος \frac{AB + AX}{BX} παίρνει τη μέγιστη τιμή του.
Να διακρίνετε δύο περιπτώσεις:
1. Το τμήμα ΑΒ είναι κάθετο στο Ρ.
2. Το τμήμα ΑΒ δεν είναι κάθετο στο Ρ.

3. Τρίτη 9 Νοεμβρίου 2010
Σε μία πόλη στις δημοτικές εκλογές του έτους α, ψήφισαν 10000 άτομα, υπήρχαν οι συνδυασμοί Α, Β, Γ, Δ
με ψήφους και ποσοστά, όπως φαίνονται στον πίνακα που ακολουθεί:
Α: ψήφοι 4000, ποσοστό 40%
Β: ψήφοι 3000, ποσοστό 30%
Γ: ψήφοι 2000, ποσοστό 20%
Δ: ψήφοι 1000, ποσοστό 10%
Το έτος α + 4 στην ίδια πόλη ψήφισαν 8000 άτομα (υπήρχε αποχή), πάλι υπήρχαν οι τέσσερις συνδυασμοί Α, Β, Γ, Δ
με ψήφους και ποσοστά, όπως φαίνεται στον παρακάτω πίνακα:
Α: ψήφοι 3000, ποσοστό 37,5%
Β: ψήφοι 2500, ποσοστό 31,25%
Γ: ψήφοι 2000, ποσοστό 25%
Δ: ψήφοι 500, ποσοστό 6,25%
Οι συνδυασμοί Β και Γ πανηγύριζαν, αφού αύξησαν τα ποσοστά τους, ο Β από 30% σε 31,25% και ο Γ από 20% σε 25%.
Ένας μαθητής ρώτησε, πώς γίνεται αυτό; μπορεί να χάνεις σε ψήφους και να αυξάνεται το ποσοστό σου σε σχέση με τις προηγούμενες εκλογές;
Μία μαθήτρια ρώτησε, πώς γίνεται να έχεις ακριβώς τους ίδιους ψήφους σε σχέση τις προηγούμενες εκλογές και τώρα να αυξάνεται το ποσοστό σου;
Να μία άσκηση για να προβληματιστούν μαθητές των τελευταίων τάξεων του Δημοτικού και των πρώτων τάξεων του Γυμνασίου, διότι υποθέτω ότι η απάντηση σε τέτοια ερωτήματα για ενήλικες είναι "προφανής".

4. Δευτέρα 7 Φεβρουαρίου 2011
Δεν γνωρίζω αν το θέμα έχει συζητηθεί παλαιότερα.
Να εξεταστεί αν υπάρχει μονότονη και συνεχής συνάρτηση στο R,
για την οποία να ισχύει: x^{2} \leq f(x) \leq x^{2} + 1 , για κάθε πραγματικό αριθμό x.5.

5. Δευτέρα 31 Ιανουαρίου 2011
Με αφορμή το θέμα που τέθηκε εδώ  viewtopic.php?f=50&t=10783
διατυπώνω το εξής κριτήριο για ισόπλευρα τρίγωνα.
Αν στο οξυγώνιο τρίγωνο ΑΒΓ φέρουμε τις εσωτερικές διχοτόμους ΑΔ, ΒΕ, ΓΖ, οι οποίες τέμνονται στο σημείο Ο και ισχύει (ΑΖΟ) = (ΒΟΔ) = (ΓΟΕ), τότε το ΑΒΓ είναι τρίγωνο ισόπλευρο.

6. Δευτέρα 31 Ιανουαρίου 2011
Με αφορμή το θέμα που έθεσε ο Θ. Μάγκος εδώ viewtopic.php?f=50&t=10783, διατυπώνω ένα αντίστοιχο ερώτημα:
Στο οξυγώνιο τρίγωνο ΑΒΓ, φέρουμε τα ύψη ΑΔ, ΒΕ, ΓΖ, τα οποία τέμνονται στο Ο.
Αν ισχύει (ΑΖΟ) = (ΕΟΓ) ή (ΕΟΓ) = (ΟΔΒ) ή (ΟΔΒ) = (ΑΖΟ), τότε το τρίγωνο ΑΒΓ είναι ισοσκελές;

7. Πέμπτη 27 Ιανουαρίου 2011
Αν x, α, β, γ > 0, τότε το παρακάτω σύστημα
x(x + α) = β(x + γ)
x(x + β) = γ(x + α)
x(x + γ) = α(x + β)
έχει και άλλες λύσεις εκτός από την x = α = β = γ;

8. Τρίτη 25 Ιανουαρίου 2011
Έχουμε μία ζυγαριά με δύο δίσκους.
Πόσα ζύγια (ακέραιου αριθμού κιλών) χρειαζόμαστε για να ζυγίσουμε
οποιοδήποτε αντικείμενο (πάλι ακέραιου αριθμού κιλών) από 1 έως και 20 κιλά; Εννοείται ότι πρέπει να βρούμε τον μικρότερο αριθμό από τα απαραίτητα ζύγια.

9. Κυριακή 23 Ιανουαρίου 2011
Είναι θέμα που το έδωσε ο Νίκος Ζανταρίδης,
Να αποδειχθεί ότι δεν υπάρχει συνεχής συνάρτηση f: R --> R,
για την οποία να ισχύει \left|f(x) - f(y) \right|\geq \sqrt{\left|x-y \right|} για κάθε x, y.

10. Σάββατο 22 Ιανουαρίου 2011
Δίνεται ο κύκλος (Κ, R) με συντεταγμένες του σημείου Κ, (R, R) και R >0.
Από σημείο Α του (Κ, R) φέρουμε την εφαπτομένη ευθεία, η οποία τέμνει τον άξονα xx' στο Β. Ονομάζουμε Γ την προβολή του Α στον άξονα yy'.
Αν Μ το μέσο του ΒΓ, να βρεθεί ο γεωμετρικός τόπος των μέσων M,
όταν το Α κινείται στον (Κ, R).
Αν υπάρχει ενδιαφέρον για το θέμα, θα δώσω και μία πρόχειρη εικόνα του γεωμετρικού τόπου. Στην υγειά του Geogebra που "μου άνοιξε τα μάτια".

11. Σάββατο 22 Ιανουαρίου 2011
Αν για τους πραγματικούς αριθμούς x_{i}, y_{i}, i= 1, 2, 3 ισχύει:
x^{2}_{1} + y^{2}_{1} = x^{2}_{2} + y^{2}_{2} = x^{2}_{3} +y^{2}_{3}=k^{2} > 0, και τα διατεταγμένα ζεύγη (x, yi) δεν ταυτίζονται,
τότε η παράσταση Α =\left|\left(x_{3}-x_{1} \right)\left(y_{2}-y_{1} \right)-\left(x_{2}-x_{1})\left(y_{3}-y_{1} \right) \right|
παίρνει τη μέγιστη τιμή της, όταν ισχύει:
\left(x_{1}-x_{2} \right)^{2} + \left(y_{1}-y_{2} \right)^{2} = \left(x_{1}-x_{3} \right)^{2} + \left(y_{1} -y_{3}\right)^{2} = \left(x_{2}-x_{3} \right)^{2}+\left(y_{2}-y_{3} \right)^{2}.
Επίσης, να δοθεί γεωμετρική ερμηνεία των ενεργειών και των συμπερασμάτων σας.

12. Πέμπτη 20 Ιανουαρίου 2011
Παρουσιάζω ένα θέμα που έχει προκύψει από τη σχολική τάξη.
Δίνεται κύκλος (Ο, ρ), μία διάμετρός του ΑΒ και σημείο Γ της ευθείας ΑΒ προς το μέρος του Β ώστε ΒΓ = 3ρ.
1) Να βρεθεί η δύναμη του σημείου Γ ως προς τον (Ο, ρ).
2) Να αποδειχθεί ότι υπάρχει σημείο Δ του (Ο, ρ), ώστε ΓΔ = 2ρ.
3) Να αποδειχθεί ότι η ευθεία ΓΔ τέμνει τον (Ο, ρ) και σε ένα άλλο σημείο έστω Ε.
4) Να βρεθεί το μήκος του ΔΕ.
Σημείωση: Το διδακτικό ενδιαφέρον βρίσκεται στο ερώτημα 4).
Να γραφούν σχόλια σχετικά με την απάντησή σας στο ερώτημα αυτό.

13. Τρίτη 11 Ιανουαρίου 2011
Να βρεθεί ο ελάχιστος αριθμός ζυγίσεων που απαιτούνται
(σε ζυγαριά με δύο δίσκους, που δέχονται "μεγάλα" βάρη)
για να εντοπίσουμε μία σφαίρα, που μόνο αυτή είναι βαρύτερη, από ένα πλήθος ν σφαιρών.

14. Τετάρτη 19 Ιανουαρίου 2011
Ως συνέχεια του θέματος που είχα αναρτήσει εδώ: viewtopic.php?f=49&t=12415, δίνω το εξής πρόβλημα.
Από ένα σύνολο ν σφαιρών, δύο είναι βαρύτερες.
Να βρεθεί ο ελάχιστος αριθμός ζυγίσεων (με ζυγό με δύο δίσκους)
που απαιτούνται για τον εντοπισμό των δύο βαρύτερων σφαιρών.

Διευκρίνιση: Οι δύο βαρύτερες σφαίρες έχουν την ίδια μάζα.

15. Δευτέρα 17 Ιανουαρίου 2011
Αν x, y, z, n, φυσικοί αριθμοί με n > 1 και ισχύει  x^{n}+y^{n}=z^{n},  τότε να αποδειχθεί ότι οι αριθμοί x, y, z είναι μεγαλύτεροι από τον n.
Σημείωση: Δεν είναι κατασκευή δική μου, την αντέγραψα από βιβλίο.

16. Σάββατο 15 Ιανουαρίου 2011
Με αφορμή το 3ο θέμα του διαγωνισμού Ευκλείδης 2011 της ΕΜΕ για την Γ Λυκείου, θέτω για απόδειξη το ακόλουθο κριτήριο.
Αν οι κύκλοι (Κ, ρ) και (Λ, R) έχουν κοινή χορδή την ΑΒ,
για ένα σημείο Γ του (Κ, ρ) και ένα σημείο Δ του (Λ, R)
ισχύει γωνία ΑΓΒ = γωνία ΑΔΒ, τότε οι κύκλοι είναι ίσοι.


17. ...................

18. Πέμπτη 6 Ιανουαρίου 2011
Το θέμα που προτείνω δεν επιχείρησα να το λύσω,
διότι με το Geogebra "είδα" ότι το ζητούμενο σημείο κινείται σε μία άγνωστη καμπύλη.
Είναι μία (μερική) γενίκευση των προβλημάτων που έθεσα http://www.mathematica.gr/forum/viewtopic.php?f=20&t=11890 και
http://www.mathematica.gr/forum/viewtopic.php?f=22&t=11967.
Δίνονται δύο ίσοι κύκλοι (Κ, ρ) και (Λ, ρ) με ΚΛ > 2ρ
και ένα σημείο Α στη μεσοκάθετο του ΚΛ.
Να βρεθεί ο γεωμετρικός τόπος των κορυφών Δ του ορθογωνίου ΑΒΓΔ,
όταν το Β ανήκει στον (Κ, ρ) και το Γ ανήκει στον (Λ, ρ).


19. Δευτέρα 10 Ιανουαρίου 2011
Αν η σύνθεση fofof είναι συνεχής για κάθε x πραγματικό, μπορεί να fof να είναι ασυνεχής σε κάποιο xο;

20. Δευτέρα 10 Ιανουαρίου 2011
Να αποδειχθεί ότι ο αριθμός 1 + n + n^{2}+n^{3} + n^{4}
δεν διαιρείται από τον 4 για καμία τιμή του φυσικού n.
Το θέμα αυτό προτάθηκε από τον Ivan Matic για τον διαγωνισμό ΒΑΜΟ
κατά την προετοιμασία στο Berkeley Mathematics Circles.
Το προτείνουμε για την προετοιμασία όσων μαθητών θα συμμετέχουν
στον διαγωνισμό "Ευκλείδη 2010" το Σάββατο 15 Ιανουαρίου 2011.

21. Κυριακή 9 Ιανουαρίου 2011
Αν για τις συνεχείς συναρτήσεις f και g ισχύει f(g(x)) = 0 και g(f(x)) = 0,
για κάθε x πραγματικό αριθμό, τότε τουλάχιστον μία από αυτές τις συναρτήσεις είναι η μηδενική;

22. Σάββατο 8 Ιανουαρίου 2011
Αν για τις συναρτήσεις f και g ισχύουν: f(g(x)) = x και g(f(x)) = x, για κάθε x πραγματικό αριθμό, τότε οι f, g είναι μεταξύ τους αντίστροφες;

23. Κυριακή 9 Ιανουαρίου 2011
Αν για τις συναρτήσεις f και g ισχύει f(g(x)) = 0, για κάθε x πραγματικό αριθμό, τότε τουλάχιστον μία από αυτές τις συναρτήσεις είναι η μηδενική;

24. Παρασκευή 7 Ιανουαρίου 2011
Αν για τις συναρτήσεις f και g ισχύει f(g(x)) = x, για κάθε x πραγματικό αριθμό, τότε οι f, g είναι μεταξύ τους αντίστροφες;

25. Παρασκευή 7 Ιανουαρίου 2011
Να αποδειχθεί ή να απορριφθεί η εξής εικασία:
Αν οι συναρτήσεις f και g είναι 1-1 και ισχύει f(g(x)) = g(f(x)) για κάθε x πραγματικό αριθμό, τότε οι συναρτήσεις αυτές είναι μεταξύ τους αντίστροφες.

26. Πέμπτη 6 Ιανουαρίου 2011
Για το πρόβλημα της 4ης κορυφής ενός ορθογωνίου.
Το κείμενο αυτό συνοψίζει και επεκτείνει τη συζήτηση και τα συμπεράσματα που προέκυψαν μετά την ανάρτηση των θεμάτων
viewtopic.php?f=20&t=11890 και viewtopic.php?f=22&t=11967
Το πρόβλημα στη γενική του μορφή διατυπώνεται ως εξής:
Δίνεται κύκλος (Κ, ρ) και σημείο Α. Αν τα σημεία Β, Γ ανήκουν στον (Κ, ρ)
και το ΑΒΓΔ είναι ορθογώνιο, να βρεθεί ο γεωμετρικός τόπος του σημείου Δ.
Το πρόβλημα αυτό το έχω δει αρκετές φορές σε προβλήματα γεωμετρικών τόπων.
Δύο πρόσφατες «ματιές» μου έπεσαν στα εξής βιβλία:
«Ευκλείδειος Γεωμετρία» για την Α’ τάξη Λυκείου Θετικής Κατευθύνσεως
του Σπύρου Κανέλλου, ΟΕΔΒ, Αθήνα, 1976, σελίδα 29.
«Straight Lines and Curves» των N. Vasilyev και V. Gutenmacher, εκδόσεις Mir, Μόσχα, 1980, σελίδα 49, πρόβλημα 2.13. Ένα εξαιρετικό βιβλίο για θέματα κινούμενων σχημάτων, στα οποία περιλαμβάνονται και οι αποκαλούμενοι γεωμετρικοί τόποι.
Για το πρόβλημα πρέπει να διακρίνουμε τρεις διαφορετικές περιπτώσεις:
1) Το σημείο Α είναι εξωτερικό του κύκλου (Κ, ρ)
2) Το σημείο Α είναι εσωτερικό του κύκλου (Κ, ρ)
3) Το σημείο Α ανήκει στον κύκλο (Κ, ρ)
Επίσης, για τα σημεία Β και Γ πρέπει να διακρίνουμε δύο διαφορετικές διατάξεις τις ΑΒΓΔ και ΑΒΔΓ για κάθε μία από τις 3 περιπτώσεις.
Στη διαδικασία επίλυσης του προβλήματος συμμετείχαν με λύσεις, ιδέες και προτάσεις οι, p-gianno, rek2, Σωτήρης Λουρίδας και Κώστας Δόρτσιος.
Μάλιστα ο Κώστας πρότεινε την εξής επέκταση του προβλήματος:
Θα μπορούσαμε να ζητήσουμε και το γ.τ. του κέντρου Σ του ορθογωνίου ΑΒΓΔ.
Η λύση στην πρώτη περίπτωση είναι τόξο κύκλου και όχι κύκλος όπως «φαίνεται» με μια πρώτη ματιά. Αυτό συμβαίνει επειδή το σημείο Δ δεν μπορεί να κατέχει όλες τις θέσεις του ομόκεντρου κύκλου προς τον (Κ, ρ).
Η λύση στη δεύτερη περίπτωση είναι το σημείο Δ κινείται σε κύκλο ομόκεντρο του (Κ, ρ) του οποίου την ακτίνα μπορούμε να προσδιορίσουμε αξιοποιώντας το 1ο θεώρημα των διαμέσων ενός τριγώνου. Ακόμα και όταν τα σημεία Β και Γ έχουν διαφορετική διάταξη στο ορθογώνιο, πάλι η λύση είναι κύκλος ομόκεντρος προς τον αρχικό.
Στην τρίτη περίπτωση που είναι η πιο απλή, το σημείο Δ είναι και αυτό σημείο του (Κ, ρ).
Μάλιστα, αν η διάταξη των κορυφών του ορθογωνίου είναι ΑΒΓΔ, τότε το Δ είναι το αντιδιαμετρικό οποιουδήποτε σημείου του (Κ, ρ) θεωρήσουμε ως Β.
Αν η διάταξη είναι ΑΒΔΓ, τότε το Δ είναι το αντιδιαμετρικό του Α, δηλαδή είναι μοναδικό.
Τι το ενδιαφέρον έχει το πρόβλημα αυτό από την άποψη της διδασκαλίας της Γεωμετρίας;
Για την Α’ Λυκείου οι περιπτώσεις 1η και 3η είναι κατάλληλες για μελέτη από τους μαθητές της τάξης αυτής, αφού οι γνώσεις που απαιτούνται είναι οι ιδιότητα της μεσοκαθέτου, ο ορισμός του κύκλου, οι ιδιότητες του ορθογωνίου. Επιπλέον, αυτό είναι το βασικό χαρακτηριστικό του προβλήματος, οι μαθητές πρέπει να γίνουν προσεκτικοί ώστε να διακρίνουν ακραίες περιπτώσεις, περιορισμούς στο πρόβλημα. Επίσης, ο διδάσκων μπορεί να θέσει την περίπτωση 2η, η οποία δεν αντιμετωπίζεται από μαθητές της Α Λυκείου, λόγω του θεωρήματος των διαμέσων που πρέπει να γνωρίζουν, ως δείγμα ότι ένα πρόβλημα με μία τροποποίηση των δεδομένων δυσκολεύει δραματικά.
Επίσης, για τους μαθητές της Θετικής και Τεχνολογικής Κατεύθυνσης της Β΄ Λυκείου, το θέμα έχει ενδιαφέρον να αντιμετωπιστεί και με διανυσματικό τρόπο και να συγκριθεί η τεχνική αυτή,με αυτές της Ευκλείδειας Γεωμετρίας. Ας σημειωθεί ότι ένα τέτοιο πρόβλημα περιέχει και «οριακές καταστάσεις», όπως επεσήμανε ο Σ. Λουρίδας, για παράδειγμα, τι θα γίνει αν το σημείο Β πλησιάζει πολύ κοντά στο Α, ή συμπέσει με αυτό;
Ιδιαίτερη διδακτική αξία έχει ένα τέτοιο πρόβλημα, διότι θέτει νέα ερωτήματα με ελαφρές έως ριζικές τροποποιήσεις των δεδομένων και των ζητουμένων του, όπως έδειξε ο Κ. Δόρτσιος (και θα κάνω κι εγώ), και κυρίως διότι μπορεί να μπλέξει στο «παιχνίδι της έρευνας» και μαθητές που δεν έχουν καλό μαθηματικό υπόβαθρο, αλλά μπορούν να χειριστούν υπολογιστικά εργαλεία, όπως το Geogebra, Cabri, ή ακόμα κι αν δεν έχουν ιδέα από όλα αυτά, αρκεί να έχουν το ενδιαφέρον να παρακολουθήσουν τη διαδικασία «ψαξίματος» της λύσης του προβλήματος σε μία οθόνη όταν ο διδάσκων ή κάποιος συμμαθητής τους χειρίζεται τα εργαλεία του προγράμματος.
Τέτοιου είδους προβλήματα δείχνουν πώς λειτουργούν και εμπνέονται οι κατασκευαστές μαθηματικών προβλημάτων, πώς δοκιμάζουν με υπολογιστικά μέσα τις εικασίες τους και όταν βεβαιωθούν για αυτές δίνουν «το οριστικό κτύπημα», δηλαδή την απόδειξη.
Θέτω και εγώ με τη σειρά μου ένα πρόβλημα σαφώς δυσκολότερο, το οποίο θα αναρτήσω σύντομα.

27. Κυριακή 19 Δεκεμβρίου 2010
Να βρείτε όλους τους πρώτους φυσικούς αριθμούς α, β και γ για τους οποίους ισχύει:   αβ + βγ + αγ > αβγ.
Την πηγή από όπου πήρα το πρόβλημα, θα την αναρτήσω μετά την επίλυση της άσκησης από τους ενδιαφερόμενους νεαρούς λύτες.

28. Κυριακή 26 Δεκεμβρίου 2010
Ως συνέχεια του θέματος http://www.mathematica.gr/forum/viewtopic.php?f=20&t=11890 προτείνω
το εξής θέμα:
Δίνεται κύκλος (Κ, ρ) και σημείο Α στο εσωτερικό του.
Αν τα σημεία σημεία Β, Γ ανήκουν στον (Κ, ρ) και το ΑΒΓΔ είναι ορθογώνιο, να βρεθεί ο γεωμετρικός τόπος των σημείων Δ.

Μετά τη δημοσίευση λύσεων στο θέμα αυτό, θα κάνω μία σειρά παρατηρήσεων, διάκρισης περιπτώσεων και σχετικών σχολίων.

29. Τετάρτη 22 Δεκεμβρίου 2010
Δίνεται ένα τρίγωνο ΑΒΓ. Να κατασκευαστεί τετράγωνο του οποίου οι κορυφές είναι σημεία του τριγώνου ΑΒΓ.  (Εγγεγραμμένο τετράγωνο σε τρίγωνο).

30. Τετάρτη 22 Δεκεμβρίου 2010
Δίνεται το πολυώνυμο
P(x) = x^{4}+ (a-1)x^{3} - (2a^{2}+1)x^{2}+ (a^{2}+1)x + a^{2}-a.
Να αποδειχθεί ότι για κάθε τιμή της παραμέτρου a, το πολυώνυμο έχει πραγματικές ρίζες.

31. Δευτέρα 20 Δεκεμβρίου 2010
Να αποδειχτεί ότι δεν υπάρχει τρίγωνο στο οποίο ο εγγεγραμμένος κύκλος
τριχοτομεί μία διχοτόμο του τριγώνου.

32. Πέμπτη 16 Δεκεμβρίου 2010
Στο συνημμένο σχήμα έχουμε ορθοκανονικό σύστημα αξόνων και ΟΒ = 10.
Το σημείο Γ κινείται στον άξονα xx', ώστε 0 < ΟΓ < 10.
Αν Δ το ίχνος της διχοτόμου της γωνίας ΒΓΟ στον άξονα yy',
να βρεθεί η απόσταση ΟΔ για τις διάφορες θέσεις του σημείου Γ.

33. Τετάρτη 15 Δεκεμβρίου 2010
Με αφορμή το θέμα που δημοσιεύθηκε εδώ http://www.mathematica.gr/forum/viewtopic.php?f=23&t=11535,  να αποδειχθεί με τη χρήση του Διανυσματικού Λογισμού το αντίστροφο.
"Αν σε ένα τρίγωνο ΑΒΓ το ΑΔ είναι ύψος του και ισχύει
 \overrightarrow{{\rm{A}}\Delta} ^{2} = \overrightarrow{{\rm{B}}\Delta}\cdot\overrightarrow{\Delta \Gamma} ,  τότε το τρίγωνο είναι ορθογώνιο με ορθή γωνία την Α".

34. Πέμπτη 16 Δεκεμβρίου 2010
Δίνεται το ισόπλευρο τρίγωνο ΑΒΓ και τα κανονικά πεντάγωνα ΑΓΔΕΖ και ΑΗΘΙΒ, τα οποία έχουν κοινά σημεία με το τρίγωνο ΑΒΓ μόνο αυτά των πλευρών ΑΓ και ΑΒ αντίστοιχα.
1) Να εξεταστεί αν ΗΒ // ΖΓ.
2) Να εξεταστεί αν τα σημεία Θ, Α, Δ είναι συνευθειακά
3) Να εξεταστεί αν ΘΕ // ΒΓ.
4) Να εξεταστεί αν το τετράπλευρο ΗΖΓΒ είναι εγγράψιμο σε κύκλο.

35. Πέμπτη 9 Δεκεμβρίου 2010
Με αφορμή το θέμα που τέθηκε http://www.mathematica.gr/forum/viewtopic.php?f=52&t=11461,
θέτω ένα σχετικό ερώτημα:
Αν δύο συναρτήσεις f, g είναι ασυνεχείς σε κάθε σημείο του R,
τότε και η σύνθεση fog θα είναι και αυτή ασυνεχής στο R;

36. Σάββατο 4 Δεκεμβρίου 2010

Για τη συνεχή συνάρτηση f στο [0, 1] ισχύει:
\int_{0}^{x}{f(t)dt} = 1 - \frac{1}{g(x)} για κάθε x του διαστήματος [0, 1]
και η g είναι μία άγνωστη συνάρτηση ορισμένη στο ίδιο διάστημα.
Να αποδειχθεί ότι: g(0) + g(1/2) + g(1) > 0.

37. Σάββατο 4 Δεκεμβρίου 2010
Με αφορμή του θέμα που παρουσιάστηκε εδώ,
http://www.mathematica.gr/forum/viewtopic.php?f=22&t=11119
να κατασκευαστεί τρίγωνο ΑΒΓ με τα εξής δεδομένα:
ΒΓ = 8, γωνία ΒΓΑ = 120 μοίρες και η διάμεσος ΓΔ είναι κάθετη στην πλευρά ΒΓ.

38. Κυριακή 28 Νοεμβρίου 2010
Με αφορμή το θέμα, posting.php?f=51&t=11104, προτείνω μία τροποποίηση που το δυσκολεύει αρκετά. Η δική μου προσέγγιση απαιτεί και μέγιστα-ελάχιστα με τη βοήθεια των παραγώγων.
Έστω οι μιγαδικοί z με z = (\lambda ^{2}-3) + (2λ+1)i , λεR
1) Να βρείτε τον γεωμετρικό τόπο των εικόνων των μιγαδικών z,
2) Να βρείτε τον μιγαδικό z που η εικόνα του βρίσκεται πλησιέστερα στην αρχή των αξόνων.
Έχει δίκιο ο Κώστας όπως έμμεσα και ευγενικά το τόνισε.
Δεν είναι εύκολο χωρίς τον τύπο επίλυσης τριτοβάθμιων πολυωνυμικών εξισώσεων να βρούμε τη ζητούμενη λύση.
Το ερώτημα 2) έπρεπε να τροποποιηθεί ως εξής:
Να αποδείξετε ότι υπάρχει ένας μοναδικός μιγαδικός z που η εικόνα του βρίσκεται πλησιέστερα στην αρχή των αξόνων.
Εξάλλου, για τον λόγο αυτό είχα αναφέρει και τον εντοπισμό ακροτάτων με τη βοήθεια των παραγώγων.

39. Παρασκευή 26 Νοεμβρίου 2010
Για τους πραγματικούς αριθμούς k_{i} ισχύει 0 < k_{i-1} < k_{i} για κάθε i από 1 έως n.
Να αποδειχθεί ότι η εξίσωση (ως προς τον πραγματικό αριθμό x)
(k_{n})^{x} = (k_{n-1})^{x} + (k_{n-2})^{x} + ...+(k_{2})^{x} + (k_{1})^{x}
έχει μία ακριβώς λύση.

40. Τετάρτη 13 Οκτωβρίου 2010
Σε έναν κύκλο φέρουμε τις κάθετες χορδές ΑΒ και ΓΔ.
Το μέρος του κυκλικού δίσκου που ορίζεται από τις κάθετες χορδές και το έλασσον τόξο ΑΓ έχει εμβαδόν 4 τ.μ.
Το μέρος του κυκλικού δίσκου που ορίζεται από τις κάθετες χορδές και το έλασσον τόξο ΑΔ έχει εμβαδόν 10 τ.μ.
Το μέρος του κυκλικού δίσκου που ορίζεται από τις κάθετες χορδές και το έλασσον τόξο ΒΓ έχει εμβαδόν 8 τ.μ.
Μπορεί να βρεθεί το εμβαδόν του μέρους του κυκλικού δίσκου που ορίζεται από τις κάθετες χορδές και το έλασσον τόξο ΒΔ;

41. Σάββατο 13 Νοεμβρίου 2010
Με αφορμή το θέμα που προτάθηκε εδώ http://www.mathematica.gr/forum/viewtopic.php?f=51&t=10661
να απαντηθούν τα παρακάτω ερωτήματα:
Ποια είναι η σχέση της εικόνας ενός μιγαδικού z και της εικόνας
1. του -z
2. του -\bar{z}
3. του 1/z
4. του -1/\bar{z}
5. του z^{2}
6. του -1/z
7. του \left(\bar{z} \right)^{2}
Καθώς η εκφώνηση ζητάει τη σχετική θέση των εικόνων των μιγαδικών, καταλαβαίνω ότι δεν ενδιαφερόμαστε για τα τεταρτημόρια στα οποία βρίσκονται οι εικόνες.
Ας είναι \displaystyle{z=x+yi} με \displaystyle{x,y \in \mathbb{R}.}
Επειδή \displaystyle{-z=-x-yi}, οι \displaystyle{z,-z} έχουν εικόνες σημεία συμμετρικά ως προς την αρχή του μιγαδικού επιπέδου.
Επειδή \displaystyle{-\bar{z}=-x+yi}, οι \displaystyle{z,-\bar{z}} έχουν εικόνες σημεία συμμετρικά ως προς τον άξονα των φανταστικών.
Για τα υπόλοιπα, χρειαζόμαστε τον μιγαδικό υπό τριγωνομετρική μορφή.
Θάνο,
αυτό ακριβώς ήθελα να τονίσω. Την αξία της χρήσης της τριγωνομετρικής μορφής ενός μιγαδικού για τον πολλαπλασιασμό (ή τη διαίρεση) και κατ΄ επέκταση τη χρήση της στις δυνάμεις μιγαδικών.
Και μάλιστα, όταν αυτό είναι πλήρως κατανοητό από τον καθένα.
Για παράδειγμα. Αν ο z σε τριγωνομετρική μορφή γράφεται: z = \left|z \right| \left(cos(\phi) + isin\phi  \right), τότε \frac{1}{z} = z^{-1}= \left|z \right|^{-1}\left(cos(-\phi) + isin(-\phi) \right).
Γνωρίζουμε ότι οι εικόνες των αριθμών z και -z είναι συμμετρικές ως προς το σημείο (0, 0). Άρα, οι εικόνες των -z και 1/z και (0, 0) είναι σημεία συνευθειακά και η απόσταση του z από το (0, 0) είναι αντιστρόφως ανάλογη με αυτή του 1/z ως προς το (0, 0).
Επίσης, μπορούμε να αναφερθούμε και σε στροφή των εικόνων των z και 1/z γύρω από το (0, 0) με μία μεταβολή της απόστασης τους από αυτό.
Αντίστοιχα, οι θέσεις των εικόνων του z και του z^{2}, σχετίζονται με μία έννοια "στροφής" του δεύτερου προς τα αριστερά με κέντρο στροφής το (0, 0) κατά γωνία φ, όπου φ είναι η γωνία που σχηματίζει η διανυσματική ακτίνα της εικόνας του z σε σχέση με τον άξονα των πραγματικών αριθμών.
Οι τριγωνομετρική μορφή ενός μιγαδικού (πολικές συντεταγμένες) ερμηνεύει (προφανώς) και τη θέση των εικόνων των -z, \bar{z}, κλπ σε σχέση με την εικόνα του z, όπως γνωρίζουμε και με τη χρήση καρτεσιανών συντεταγμένων.
Ειλικρινά, δεν βλέπω κάποιο σοβαρό λόγο να υπάρχουν στοιχεία μιγαδικών αριθμών στην σχολική ύλη, χωρίς την τριγωνομετρική τους μορφή, αφού αυτή είναι που αναδεικνύει την δυναμική της στην επίλυση εξισώσεων και στην εκτέλεση φαινομενικά "δύσκολων" πράξεων.
Συνεπώς, αφού διώξαμε την Τριγωνομετρία (θα γελάσουμε πολύ του χρόνου), δεν βλέπω κάποιον λόγο να κρατήσουμε κάποια στοιχεία μιγαδικών αριθμών.
Δεν προσθέτουν κάτι στην μαθηματική μόρφωση των μαθητών μας. Είναι μία "δήθεν" γνώση χωρίς ουσία και στόχο. Κατανοώ ότι αυτή είναι μία λίγο "ακραία" άποψη, είναι όμως άποψη που κατατίθεται δημόσια και για το λόγο αυτό είναι ειλικρινής.

42. Δευτέρα 27 Σεπτεμβρίου 2010
Με αφορμή την άσκηση του Chris php?f=51&t=9494 για ένα θέμα με μέτρα μιγαδικών δίνω το εξής:
Για κάθε φυσικό n μεγαλύτερο ή ίσο του 2, να αποδειχθεί ότι
 1 \leq  (n-1)sin(\frac{\pi }{n}).

43. Πέμπτη 23 Σεπτεμβρίου 2010
Το θεώρημα αυτό, το βρήκα σε έναν υποφάκελλο στο πρόγραμμα Geometer's Sketchpad, με το τίτλο "Το θεώρημα του Blackwell".
Πράγματι, υπάρχει ως τέτοιο θεώρημα στο βιβλίο του Blackwell - ο τίτλος είναι σε μετάφραση -  "Η Γεωμετρία στην Αρχιτεκτονική" , εκδόσεις Key Curriculum Press, Ιούνιος 1984. Η εκφώνηση του θεωρήματος είναι η εξής:
"Σε κάθε ορθογώνιο τρίγωνο, τα ημικύκλια που έχουν διάμετρο τις πλευρές του τριγώνου και δεν έχουν άλλα κοινά σημεία με αυτό,
εφάπτονται σε ένα τετράγωνο που έχει πλευρά την ημιπερίμετρο του τριγώνου".
Ζητείται η απόδειξη του θεωρήματος.

44. Τρίτη 14 Σεπτεμβρίου 2010
Η άσκηση αυτή βασίζεται στην άσκηση που πρότεινε ο Μιχάλης Νάννος "βρείτε τη γωνία χ(42)" και στην προσέγγισή της από τον Σεραφείμ Τσιπέλη.
Να αποδειχθεί η σχέση:
\frac{sin48.sin18}{sin(48+18)} + \frac{sin42.sin54}{sin(42+54)} + \frac{sin12.sin6}{sin(12+6)} = \frac{\sqrt{3}}{2}.

45. Παρασκευή 16 Ιουλίου 2010
Το ερώτημα που θέτω είναι μέρος ενός γενικότερου προβλήματος που τέθηκε στο Φόρουμ και για το οποίο ψάχνω μία πλήρη λύση.
Το ερώτημα ήταν: "Να κατασκευαστεί ευθεία, που να χωρίζει ένα τετράπλευρο σε δύο ισεμβαδικά και ταυτόχρονα ισοπεριμετρικά σχήματα". Είναι το πρόβλημα viewtopic.php?f=62&t=8093.
Το ερώτημα που θέτω είναι: "Να εξεταστεί αν υπάρχει ευθεία που να χωρίζει ορθογώνιο και σκαληνό τρίγωνο σε δύο σχήματα ισεμβαδικά και ισοπεριμετρικά".

46. Πέμπτη 1 Ιουλίου 2010
Στο τετράγωνο ΑΒΓΔ φέρουμε τις διαγώνιες ΑΓ και ΒΔ που τέμνονται στο Ο, και χαράσουμε το τεταρτοκύκλιο (εντός του τετραγώνου) με κέντρο Δ και ακτίνα ΔΑ. Αυτό τέμνει την ΒΔ στο σημείο Ε. Να υπολογιστεί η ακτίνα του κύκλου που εφάπτεται στο ΑΟ, στο ΟΕ και στο τόξο ΑΕ, συναρτήσει της πλευράς α του ΑΒΓΔ.

47. Πέμπτη 15 Απριλίου 2010
Δίνονται τα σημεία Α(2α, 0) Β(4α, 0) και Γ(6α, 0) με α > 0 και ένα ημικύκλιο με διάμετρο ΑΓ, κέντρο Β και ακτίνα 2α.
Φέρουμε την εφαπτομένη ευθεία ε από το (0, 0) προς το ημικύκλιο.
Προβάλλουμε κάθε σημείο Σ του ημικυκλίου στον άξονα xx' και στην ευθεία ε και ονομάζουμε τις προβολές του, Ε και Ζ αντίστοιχα.
1. Να βρεθεί εκείνο το σημείο Σ, ώστε το εμβαδόν του τετραπλεύρου ΟΕΣΖ να είναι το μέγιστο.
2. Να εξετάσετε αν υπάρχει σημείο Σ του ημικυκλίου, ώστε το εμβαδόν του τετραπλεύρου ΟΕΣΖ να ισούται με αυτό του ημικυκλίου.

48. Τρίτη 13 Απριλίου 2010
Δίνεται η συνάρτηση f με f(x) = (x^2)*ln(x) για x > 0 και f(0) = 0.
Να αποδειχθεί ότι το εμβαδόν μεταξύ της γραφικής παράστασης της f και της ευθείας y = x είναι μικρότερο του 2.

49. Δευτέρα 22 Μαρτίου 2010
Είναι μία άσκηση βασισμένη σε αυτήν που πρότεινε ο math-finder.
Δίνεται τετράγωνο ΑΒΓΔ με μήκος πλευράς α και τετράγωνο ΕΖΗΘ με μήκος πλευράς β, με α = 3β,  ώστε το δεύτερο να βρίσκεται στο εσωτερικό του πρώτου.
Να βρεθεί η θέση του ΕΖΗΘ, ώστε το άθροισμα των αποστάσεων των κορυφών του από τις πλευρές του ΑΒΓΔ να είναι το ελάχιστο δυνατό.
Η άσκηση μπορεί να γενικευτεί για α = νβ.
Η λύση της δίνει και άλλη ενδιαφέρουσα γενίκευση.

50. Πέμπτη 18 Μαρτίου 2010
Να αποδειχθεί ότι δεν υπάρχει σημείο μιας έλλειψης, από το οποίο ο μεγάλος ή ο μικρός της άξονας να "φαίνεται" υπό ορθή γωνία.

51. Κυριακή 7 Μαρτίου 2010
Δίνεται ισόπλευρο τρίγωνο ΑΒΓ και ο εγγεγραμμένος σε αυτό κύκλος, ο οποίος εφάπτεται στις πλευρές ΑΒ, ΑΓ, ΒΓ στα σημεία Δ, Ε, Ζ αντίστοιχα.
Να βρεθεί σημείο Σ της ΒΓ, ώστε η ευθεία ΑΣ να αποκόπτει από τον κύκλο, κυκλικό τμήμα ισεμβαδικό με αυτό του μικτόγραμμου χωρίου ΑΔΕ.

52. Δευτέρα 8 Μαρτίου 2010
Δίνονται δύο ίσα ισόπλευρα τρίγωνα.
Στο πρώτο σχεδιάζουμε ένα τετράγωνο που όλες οι κορυφές του είναι σημεία του τριγώνου.
Στο δεύτερο σχεδιάζουμε ένα τετράγωνο που μόνο οι τρεις κορυφές του είναι σημεία του τριγώνου. Αιτιολογήστε ποιο από τα δύο τετράγωνα έχει μεγαλύτερο εμβαδόν. 


 53. Σάββατο 27 Φεβρουαρίου 2010
Να αποδειχθεί η ανισότητα:
x(a-z) + y(a-x) + z(a-y) < a^2, αν a > 0 και x, y, z αριθμοί του διαστήματος [0, a].
Η άσκηση τέθηκε στο τελευταίο τεύχος του Ευκλείδη Β, σελίδα 48 από τον Γιώργο Τσαπακίδη και εκεί αποδεικνύεται με γεωμετρικό (άκρως ενδιαφέροντα και ελκυστικό) τρόπο.
Εδώ επιπλέον θέτουμε το ερώτημα, αν και πότε ισχύει η ισότητα στη ζητούμενη σχέση.

54. Πέμπτη 25 Φεβρουαρίου 2010
Δίνεται τετράγωνο ΑΒΓΔ. Να εξεταστεί αν υπάρχει ισόπλευρο τρίγωνο με τις κορυφές του επί των πλευρών του ΑΒΓΔ, όταν καμιά από αυτές δεν συμπίπτει με κορυφές του τετραγώνου.
ΣΗΜΕΙΩΣΗ: Η περίπτωση "εγγεγραμμένου" ισόπλευρου τριγώνου σε τετράγωνο, με τον σχετικό υπολογισμό του μήκους της πλευράς του, δόθηκε από τον Γιώργο Τσαπακίδη στο τελευταίο τεύχος του "Ευκλείδη Β", σ. 45.

55.  Κυριακή 28 Φεβρουαρίου 2010
Δίνεται ισόπλευρο τρίγωνο πλευράς α. Να εξεταστεί αν υπάρχει σημείο στο εσωτερικό του τρίγωνου αυτού (εκτός από το κέντρο βάρους του), τέτοιο ώστε το άθροισμα των αποστάσεων του από τις κορυφές του τριγώνου, να ισούται με α\sqrt{3}.
Περιορισμός: Να μην γίνει χρήση θεωρημάτων του Διαφορικού Λογισμού.

56. Πέμπτη 11 Φεβρουαρίου 2010
Να υπολογιστεί το παρακάτω όριο.
\displaystyle \lim_{x\rightarrow 0^{+}} \left(\ln(x)+\frac{1}{x}\right)

57. Παρασκευή 15 Απριλίου 2011
Δίνονται οι ευθείες: y = 0, y = 4, y = 2x, y = 2x - 8.
Πόσες ελλείψεις υπάρχουν, τέτοιες ώστε κάθε μία από τις ευθείες αυτές
να έχει ένα μόνο κοινό σημείο με την έλλειψη.
Προφανώς, το ερώτημα μπορεί να τεθεί για κάθε παραλληλόγραμμο.

58. Παρασκευή 15 Απριλίου 2011
Δίνεται η συνάρτηση f(x)= \frac{lnx}{x} με x > 0.
Να εξεταστεί αν υπάρχει εφαπτομένη ευθεία στη γραφική παράσταση της συνάρτησης f,
η οποία να την τέμνει στο σημείο (3/2, f(3/2)).

59. Κυριακή 19 Απριλίου 2011
http://www.mathematica.gr/forum/viewtopic.php?f=112&t=14887.
Το θέμα που προτείνω είναι πάλι από διαγωνισμό Πανρωσικό.
Αυτή τη φορά κοίταξα καλά όλα τα σχετικά θέματα που έχουν
προταθεί στη στήλη "Γεωμετρία διαγωνισμών για Seniors".
Αλλά ποτέ δεν πρέπει να είναι κανείς σίγουρος.
Το ζητούμενο πάλι είναι να δοθούν πολλές και διαφορετικές λύσεις.
Το θέμα είχε χωριστεί σε δύο σκέλη α) και β) όπως το γράφω.
α) Κάθε μία από τις διαγώνιες ενός κυρτού τετραπλεύρου το διαιρεί σε δύο ισεμβαδικά σχήματα. Να αποδείξετε ότι το τετράπλευρο είναι παραλληλόγραμμο.
β) Δίνεται κυρτό εξάγωνο ΑΒΓΔΕΖ. Αν κάθε μία από τις διαγώνιες ΑΔ, ΒΕ και ΓΖ το διαιρεί σε δύο ισεμβαδικά σχήματα, να αποδείξετε ότι οι τρεις αυτές διαγώνιοι έχουν ένα κοινό σημείο.

60. Τετάρτη 20 Απριλίου 2011
Δίνεται ένας κύκλος (Ο, ρ) και ένας κύκλος (Κ, α) με 5α = ρ, που εφάπτεται εξωτερικά στον (Ο, ρ). Θεωρούμε ένα σημείο Ν του μικρού κύκλου, φέρουμε το τμήμα ΟΝ
και έστω Χ το μέσο του. Αν ο κύκλος (Κ, α) στρέφεται περί τον κύκλο (Ο, ρ), να βρεθεί ο γεωμετρικός τόπος των σημείων Χ.

61. Πέμπτη 13 Ιανουαρίου 2011
Δίνονται οι κύκλοι (Κ, 4) και (Λ, 2) με ΚΛ = 6.
Να αποδειχθεί ότι υπάρχει τετράγωνο που εφάπτεται στον (Κ, 4) σε δύο ακριβώς σημεία
και στον (Λ, 2) επίσης σε δύο ακριβώς σημεία του.
Να βρεθεί το εμβαδόν αυτού του τετραγώνου.
62. Πέμπτη 21 Απριλίου 2011.
http://www.mathematica.gr/forum/viewtopic.php?f=62&t=14961
Δίνεται ένας κύκλος (Ο, ρ) και ένας κύκλος (Κ, α) με 5α = ρ, ο οποίος εφάπτεται εξωτερικά στον (Ο, ρ). Θεωρούμε ένα σημείο Ν του μικρού κύκλου, φέρουμε το τμήμα ΟΝ και έστω Χ το μέσο του. Αν ο κύκλος (Κ, α) στρέφεται περί τον κύκλο (Ο, ρ), να βρεθεί ο γεωμετρικός τόπος των σημείων Χ. Δεν ήμουν σαφής. Ζητάω την εξίσωση της καμπύλης πάνω στην οποία κινείται το σημείο.Με μια ματιά στο σχήμα που έχω αναρτήσει με το πρόγραμμα Cabri II, φαίνεται ότι το σημείο κινείται σε κλειστή καμπύλη που μοιάζει με μαργαρίτα. Εξάλλου, το θέμα υποτίθεται είναι για διδάσκοντες.

63. Πέμπτη 13 Οκτωβρίου 2011.
Δίνεται τετράγωνο ΑΒΓΔ με ΑΒ = α. Αν Ο το μέσο του ΑΒ, ονομάζουμε Ε
το σημείο τομής του κύκλου (Ο, ΟΑ) και του τμήματος ΔΟ.
Ονομάζουμε Ζ το σημείο τομής του κύκλου (Ο, ΟΑ) με τη διαγώνιο ΑΓ
και Η το σημείο τομής της ΑΓ με το ΟΔ.
1. Να υπολογιστεί το μήκος του ΕΖ συναρτήσει του α.
2. Να υπολογιστεί το εμβαδόν του τριγώνου ΕΖΗ συναρτήσει του α.

64. Σάββατο 15 Οκτωβρίου 2011.
Το ABC είναι ισόπλευρο τρίγωνο.
Ονομάζουμε M, N, τα μέσα των ημικυκλίων με διαμέτρους τις πλευρές AB, AC αντίστοιχα,
τα οποία δεν έχουν άλλα κοινά σημεία με το τριγωνικό χωρίο εκτός από τις κορυφές του.
Να εκφράσετε το μήκος MN συναρτήσει της πλευράς του ισοπλεύρου τριγώνου.
Παραγγελία: Ο τρόπος υπολογισμού να γίνει:
1. Με όμοια τρίγωνα.
2. Με Τριγωνομετρία.
3. Με Αναλυτική Γεωμετρία (αν και δεν ανήκει στον φάκελο Ευκλείδεια Γεωμετρία), είναι παραγγελία για τον Γ. Ρίζο και τους άλλους αιρετικούς, μικρούς και μεγάλους.
4. Με τη χρήση της στροφής ως προς σημείο, (και αυτό αιρετικό, αφού ανήκει στους γεωμετρικούς μετασχηματισμούς - αντιαρχαιοελληνικόν, αλλά ολίγοτερον καταραμένο σε σχέση την αίρεση του Καρτεσίου).
5. Με όποιον άλλο τρόπο μπορείτε, αρκεί να είναι σεμνός και ηθικός.

65. Πέμπτη 15 Δεκεμβρίου 2011.
http://www.mathematica.gr/forum/viewtopic.php?f=59&t=21331
Δίνεται το ισοσκελές τρίγωνο ABC με AB = AC = 2a και BC = a > 0. Το σημείο D κινείται στο μείζον τόξο BC του περιγεγραμμένου κύκλου του τριγώνου ABC. Αν E είναι το σημείο τομής της διχοτόμου της γωνίας BAD με την BD, να βρεθεί ο γεωμετρικός τόπος του σημείου E συναρτήσει του αριθμού a.

66. Κυριακή, 17 Απριλίου 2011.
http://www.mathematica.gr/forum/viewtopic.php?f=112&t=14876
Το παρακάτω θέμα είναι γνωστό. Δεν αναφέρω την πηγή του, ούτε πότε τέθηκε ως διαγωνιστικό πρόβλημα. 
Το ζητούμενο είναι να δοθεί απάντηση με όσο περισσότερους τρόπους μπορούμε.
Για το λόγο αυτό το έθεσα ως πρόβλημα των "μεγάλων", 
για υπάρχει μία ποικιλία τεχνικών και μεθόδων στην αντιμετώπισή του.
Εκτός αν έχει ξανατεθεί και δεν το ξέρω, οπότε ... 
τα λαγωνικά του Φόρουμ θα μας παραπέμψουν στο ταμείο για τα περαιτέρω.
Από το μέσο Μ της βάσης ΑΓ ενός ισοσκελούς τριγώνου ΑΒΓ 
φέρουμε την κάθετο ΜΗ στην πλευρά ΒΓ.
Το σημείο Ρ είναι το μέσο του τμήματος ΜΗ. 
Να αποδείξετε ότι ΑΗ είναι κάθετο στο ΒΡ. 


67. Σάββατο 15 Οκτωβρίου 2011
Σε μία ευθεία είναι γνωστές οι θέσεις των σημείων Α, Β και C για τα οποία ισχύουν τα εξής: Από τα σημεία A και B διέρχεται ένας κύκλος (c) και το σημείο C ανήκει στην εφαπτομένη του (c)Να βρεθεί ο γεωμετρικός τόπος των σημείων επαφής των εφαπτόμενων του (c) που διέρχονται από το C.

68.Κυριακή 29 Ιανουαρίου 2012.
Δίνεται η έλλειψη με εξίσωση \frac{x^{2}}{a^{2}}+\frac{y^{2}}{b^{2}}=1 με a > b > 0.1. Να αποδειχθεί ότι υπάρχουν μόνο δύο κύκλοι, οι οποίοι εφάπτονται στους άξονες xx' και yy'
έχουν κέντρα με θετικές συντεταγμένες και έχουν μόνο ένα κοινό σημείο με την έλλειψη.
2. Να εκφράσετε την απόσταση των κέντρων αυτών των δύο κύκλων συναρτήσει των αριθμών a, b.

Ακολουθεί σύντομα και η συνέχεια του "σήριαλ". Το φινάλε είναι γνωστό.... Ένας από τους δύο κύκλους θα παντρευτεί την έλλειψη.
Τώρα, ακολουθεί το δεύτερο μέρος του σήριαλ, όπως έχω υποσχεθεί.
Όταν το κέντρο της έλλειψης κινείται στον άξονα xx' και ο ένας άξονας συμμετρίας της είναι παράλληλος με αυτόν, να βρεθεί ο γεωμετρικός τόπος του "μικρού" κύκλου που "εφάπτεται" στους άξονες και στην έλλειψη.

69. Τετάρτη, 15 Φεβρουαρίου 2012.
Αν μία συνάρτηση με πεδίο ορισμού το R παίρνει μόνο θετικές τιμές στο R και είναι δύο φορές παραγωγίσιμη, τότε δεν μπορεί να είναι κοίλη σε όλο το R.

70. Κυριακή, 8 Ιανουαρίου 2012.
Οι πλευρές κάθε κανονικού εξαγώνου χρωματίζονται μόνο με τα χρώματα γαλάζιοκόκκινο και μαύρο, χωρίς περιορισμό στην επανάληψη των χρωμάτων.
1. Με πόσους διαφορετικούς τρόπους μπορεί να χρωματιστεί ένα κανονικό εξάγωνο;
2. Με πόσους διαφορετικούς τρόπους μπορεί να χρωματιστεί ένα σχήμα που αποτελείται από 3 ίσα εξάγωνα.

71. Πέμπτη, 29 Δεκεμβρίου 2011.
Το ορθογώνιο τρίγωνο ABC έχει AB = 1 , AC = 2 και υποτείνουσα την BC.
Το ορθογώνιο τρίγωνο BCD έχει CD = 3 και υποτείνουσα την BD.
Το ορθογώνιο τρίγωνο BDE έχει υποτείνουσα την BE, η οποία βρίσκεται στην προέκταση της AB όπως στο συνημμένο σχήμα. Να υπολογιστούν τα μήκη BE και DE.

Προφανώς, αν βρεθεί ένας τρόπος επίλυσης, αυτός βοηθά στην απάντηση και για γενικευμένα μήκη, π.χ. AB = aAC = b, κλπ.

72. Τετάρτη, 22 Δεκεμβρίου 2011.
Με αφορμή τη συζήτηση που αναπτύχθηκε εδώ:
http://www.mathematica.gr/forum/viewtopic.php?f=112&t=20919

Να αποδειχθεί με τη βοήθεια των ιδιοτήτων των συνεχών συναρτήσεων ότι το πρόβλημα που έθεσε ο καλός φίλος Νίκος Κυριαζής έχει λύση.
Το πρόβλημα είναι το εξής: 
Δίνεται μία ευθεία (a) και τα διατεταγμένα σημεία της A, C, B με AC < CB
Να αποδειχθεί ότι υπάρχει σημείο D της (a) τέτοιο ώστε να ισχύει:
\frac{AC^{2}}{CB^{2}}= \frac{AD}{DB}.

73. Πέμπτη, 5 Απριλίου 2012.
Το θέμα που παρουσιάζουμε είναι μία μερική γενίκευση του θέματος που προτάθηκε εδώ:
http://www.mathematica.gr/forum/viewtopic.php?f=20&t=24633 

Το σημείο G κινείται στην διαγώνιο AD του κανονικού εξαγώνου ABCDEF.
Αν το FGHJKL είναι και αυτό κανονικό εξαγώνο, να αποδειχθεί ότι η κορυφή J κινείται στον φορέα της πλευράς CD.
Επίσης, να αποδειχθεί ότι ο περιγεγραμμένος κύκλος του FGHJKL διέρχεται και από το σημείο D.
Ερώτηση: Πώς μπορεί να γενικευθούν τα δύο παραπάνω προβλήματα που αφορούν το τετράγωνο και το κανονικό εξάγωνο.

Αν υπάρχει γενίκευση μπορούμε να την αποκαλέσουμε θεώρημα του κινούμενου σημείου στη διαγώνιο κανονικού πολυγώνου;
Σημείωση: Δεν πρόσεξα ότι το έθεσα ως θέμα για την Β Λυκείου. Νομίζω ότι είναι κατάλληλο και για επίπεδο γνώσεων Α τάξης του Λυκείου.
Σύμφωνα με όσα γράφτηκαν στη συνέχεια για το θέμα:
http://www.mathematica.gr/forum/viewtopic.php?f=20&t=24633
το προτεινόμενο θέμα επεκτείνεται ως εξής:
Να αποδειχθεί επίσης, ότι η κορυφή του πολυγώνου - η διαδοχική αυτής που κινείται σε ευθεία - κινείται και αυτή σε ευθεία γραμμή.
Στο εξάγωνο είναι η κορυφή Κ.
Τίθεται τώρα το ερώτημα: Σε τι γεωμετρικούς τόπους κινούνται οι άλλες κορυφές του κανονικού πολυγώνου;

Προσωπικά, θεωρώ το θέμα ελκυστικό.

74. Παρασκευή, 1 Μαρτίου, 2013.
Ένα δεδομένο τετράγωνο να χωριστεί σε μικρότερα τετράγωνα έτσι ώστε:
Να χωρίζεται σε 4 τετράγωνα.
Να χωρίζεται σε τετράγωνα.
Να χωρίζεται σε 6 τετράγωνα.
....................................
Να χωρίζεται σε 20 τετράγωνα.
Να στείλετε τις λύσεις με μορφή σχήματος για κάθε μία από τις 17 περιπτώσεις.
Σας πληροφορώ ότι μία ομάδα μαθητών του Δημοτικού του Σχολείου μου (Π.Π.Σ.Π.Θ.) (4η τάξη) 
σήμερα στο μάθημα βρήκε πολλά από αυτά τα ζητούμενα.
Υπόδειξη: Το πρόβλημα δεν αναφέρει ότι τα τετράγωνα πρέπει να είναι ίσα μεταξύ τους.
20 τετραγωνα.JPG8 τετραγωνα.JPG

75. Κυριακή, 28 Απριλίου 2013.
Δίνεται το τρίγωνο ABC όπως στο συνημμένο σχήμα και σημείο F στο εσωτερικό του. 
Τα ευθύγραμμα τμήματα BD και CE τέμνονται στο σημείο F.
Αν το εμβαδόν του τριγώνου BEF ισούται με a
το εμβαδόν του τριγώνου BFC ισούται με b
το εμβαδόν του τριγώνου DFC ισούται με c
να εκφραστεί το εμβαδόν του τετραπλεύρου AEFD συναρτήσει των ποσοτήτων abc.

(Την βρήκα στο Διαδίκτυο, δεν ξέρω αν έχει προταθεί στο Φόρουμ).


76.Δευτέρα, 22 Απριλίου 2013.
Προτείνω ένα γνωστό, με την έννοια ότι υπάρχει σε αρκετά βιβλία, πρόβλημα ελαχιστοποίησης.Αν AB είναι μία χορδή κύκλου και P ένα σημείο στο τόξο AB, 
(δεν έχει σημασία αν το P βρίσκεται στο μικρότερο ή στο μεγαλύτερο τόξο), 
τότε για είναι το άθροισμα AP + PB ελάχιστο, πρέπει το σημείο P να είναι το μέσο του τόξου AB.

77. Παρασκευή, 26 Απριλίου 2013.
Για το κυρτό τετράπλευρο ABCD γνωρίζουμε τα εξής:
AB = BC , γωνία ABD = 5 , γωνία DBC = 35 , γωνία CAD = 80. (Το μέτρο όλων των γωνιών είναι σε μοίρες) 
Να βρεθούν οι γωνίες C και D του τετραπλεύρου.

78.
                            
79. Κυριακή, 14 Απριλίου 2013.
Χθες το Σάββατο στο "Καλαμαρί" στη Θεσσαλονίκη σε μία ενδιαφέρουσα ημερίδα για τις Πανελλήνιες Εξετάσεις 
ανάμεσα στα άλλα εμφανίστηκε η συνάρτηση f(x)= \frac{lnx - x}{e^{x}}
για την οποία τονίστηκε η απόδειξη ότι είναι κοίλη έχει αρκετές δυσκολίες. 
Το βράδυ στην ταβέρνα ο Νίκος Ζανταρίδης έδωσε μία απόδειξη, η οποία πράγματι σε ένα σημείο της είχε ένα τέχνασμα. 
Ζητείται μία απόδειξη ότι η δεδομένη συνάρτηση είναι κοίλη.
Ας ασχοληθούν με αυτήν πρώτα οι υποψήφιοι μαθητές για τις εξετάσεις και μετά οι υπόλοιποι.

80. Τετάρτη, 10 Απριλίου 2013.
Να λυθεί η παρακάτω εξίσωση με δεδομένο ότι xyz είναι φυσικοί αριθμοί.  x^{4} = 10^{y}+4^{2z+1}.

81. Κυριακή 21 Απριλίου 2013.
http://www.mathematica.gr/forum/viewtopic.php?f=109&t=36568
Δίνεται ο συνημμένος πίνακας αριθμών. Ζητάμε να απαντηθούν οι ακόλουθες 5 ερωτήσεις. Προφανώς, είναι ευπρόσδεκτα και νέα ερωτήματα. Τέτοιου είδους δραστηριότητες, επειδή περιέχουν από πολύ απλά έως πολύ δύσκολα (ως προς την απάντησή τους) ερωτήματα, θεωρούμε ότι είναι κατάλληλες για μαθήματα σε μαθηματικούς ομίλους στους οποίους συμμετέχει ένας αριθμός μαθητών με διαφορετικές δυνατότητες, γνώσεις, εμπειρία και τεχνική κατάρτιση στην επίλυση προβλημάτων.
Τα ερωτήματα:
1. Πόσους αριθμούς έχει η 100η γραμμή;
2. Ο αριθμός 100 σε ποια γραμμή ανήκει;
3. Ποιος είναι ο ελάχιστος αριθμός γραμμής στην οποία το ψηφίο 0 εμφανίζεται ακριβώς 10 φορές;
4. Ποιος είναι ο ελάχιστος αριθμός γραμμής στην οποία το άθροισμα των αριθμών της είναι μεταξύ 1500 και 1600;
5. Ποιος είναι ο ελάχιστος αριθμός γραμμής στην οποία συναντάμε μόνο τετραψήφιους αριθμούς;

Αριθμός γραμμής
Στήλη 1η
Στήλη 2η
Στήλη 3η
Στήλη 4η
Στήλη 5η
Στήλη 6η
1η
1





2η
2
3




3η
4
5
6



4η
7
8
9
10


5η
11
12
13
14
15

6η
16
17
18
19
20
21
7η







Το θέμα αυτό το κουβεντιάσαμε εχτές το απόγευμα στον Όμιλο Μαθηματικών του Σχολείου μου (μαθήματα από 1η Οκτωβρίου 2012, κάθε Δευτέρα 18:00 - 20:00).
Οι μαθητές της Α τάξης του Γυμνασίου έκαναν τις εξής παρατηρήσεις οι οποίες βοηθούν στην απάντηση αρκετών ερωτημάτων.
1.Οι τελευταίοι αριθμοί κάθε γραμμής είναι τρίγωνοι αριθμοί (γνωρίζουν την έννοια αυτή και ότι οι τρίγωνοι παράγονται από τον τύπο n(n+1)/2).
2. Αρχικά παρατήρησαν ότι οι πρώτοι αριθμοί σε κάθε γραμμή προέρχονται από την πρόσθεση του αριθμού της αμέσως προηγούμενης γραμμής 
με τον αριθμό της προηγούμενης γραμμής, π.χ. 11 = 7 + 4.
3.Οι τελευταίοι αριθμοί σε κάθε γραμμή προέρχονται από την πρόσθεση του αριθμού της γραμμής τους με τον τελευταίο αριθμό της αμέσως προηγούμενης γραμμής. Π.χ. 15 = 10 + 5.
4. Οι αριθμοί σε οποιαδήποτε γραμμή εκτός του τελευταίου προέρχονται από την πρόσθεση του αριθμού της αμέσως προηγούμενης γραμμής με τον αριθμό της προηγούμενης γραμμής, πχ. 19 = 14 + 5. Οι προηγούμενες παρατηρήσεις βοηθούν να βρούμε π.χ. τους πρώτους αριθμούς (με την έννοια της διάταξης) π.χ. των 15 πρώτων γραμμών οι οποί είναι οι:
1, 2, 4, 7, 11, 16, 22, 29, 37, 46, 56, 67, 79, 92, 106. Σημειώνουμε ότι οι μαθητές των δύο τελευταίων τάξεων του Δημοτικού σε όλη την Πρωτοβάθμια εκπαίδευση μπορεί να μην είναι καθόλου εξοικειωμένοι με προβλήματα Πρακτικής Αριθμητικής, αλλά έχουν μία ικανοποιητική εξοικείωση με την έννοια του "μοτίβου" - (pattern όπως λέμε στην μητρική μας γλώσσα). Αυτό το δεύτερο είναι κάτι το πολύ θετικό, το οποίο πρέπει να τονιστεί και να αξιοποιηθεί παιδαγωγικά. Οι μαθητές της προηγούμενης δεκαετίας δεν είχαν κάποια ιδέα για την έννοια του μοτίβου, η οποία από μόνη της εμπεριέχει την έννοια των "κατασκευαστικών Μαθηματικών".

82. Δευτέρα 15 Απριλίου 2013. 
http://www.mathematica.gr/forum/viewtopic.php?f=109&t=36458
Ένα θέμα που έκανα σήμερα το απόγευμα στον Όμιλο Μαθηματικών του Σχολείου μου.
Έχουμε ένα τετράγωνο. Σε μία ώρα στις πλευρές του εμφανίζονται ίσα τετράγωνα όπως στο σχήμα. Μετά από μία ώρα στις πλευρές του νέου σχήματος εμφανίζονται κι άλλα τετράγωνα όπως στο τρίτο σχήμα. Αυτή η διαδικασία συνεχίζεται κάθε μία ώρα.Ζητάμε:
1. Πόσα τετράγωνα θα έχουμε μετά από 10 ώρες;
2. Πόσα τετράγωνα θα έχουμε μετά από n ώρες;
3. Ποια είναι η περίμετρος του σχήματος μετά από 10 ώρες;
4. Ποια είναι η περίμετρος του σχήματος μετά από n ώρες;
5. Ποια είναι η ελάχιστη ακτίνα ενός κύκλου στον οποίο να "χωράει" το σχήμα μετά από 10 ώρες;
6. Ποια είναι η ελάχιστη ακτίνα ενός κύκλου στον οποίο να "χωράει" το σχήμα μετά από n ώρες;
7. Τι άλλο μπορούμε να ζητήσουμε;

83. 17 Απριλίου 2013. 
http://www.mathematica.gr/forum/viewtopic.php?f=112&t=14887
Το θέμα που προτείνω είναι πάλι από διαγωνισμό Πανρωσικό. Αυτή τη φορά κοίταξα καλά όλα τα σχετικά θέματα που έχουν προταθεί στη στήλη "Γεωμετρία διαγωνισμών για Seniors". Αλλά ποτέ δεν πρέπει να είναι κανείς σίγουρος. Το ζητούμενο πάλι είναι να δοθούν πολλές και διαφορετικές λύσεις. Το θέμα είχε χωριστεί σε δύο σκέλη α) και β) όπως το γράφω.
α) Κάθε μία από τις διαγώνιες ενός κυρτού τετραπλεύρου το διαιρεί σε δύο ισεμβαδικά σχήματα. Να αποδείξετε ότι το τετράπλευρο είναι παραλληλόγραμμο. 
β) Δίνεται κυρτό εξάγωνο ΑΒΓΔΕΖ. Αν κάθε μία από τις διαγώνιες ΑΔ, ΒΕ και ΓΖ το διαιρεί σε δύο ισεμβαδικά σχήματα, να αποδείξετε ότι οι τρεις αυτές διαγώνιοι έχουν ένα κοινό σημείο.

84. 6 Μαϊου 2013. 
http://www.mathematica.gr/forum/viewtopic.php?f=109&t=36841
Ένα τραίνο έχει n επιβάτες και πρόκειται να κάνει k ενδιάμεσες στάσεις. Με πόσους διαφορετικούς τρόπους μπορεί να κατέβουν οι επιβάτες του τραίνου στις διάφορες στάσεις; Πιθανώς, είναι γνωστό από διάφορα βιβλία, έχει όμως ενδιαφέρον η τεχνική ή οι τεχνικές επίλυσής του.

85. 15 Ιουλίου 2013. 
http://www.mathematica.gr/forum/viewtopic.php?f=111&t=38411
Θεωρούμε τους ακέραιους αριθμούς από το 1000 έως και το 1.000.000. Πόσοι από αυτούς περιέχουν την τριάδα 123 ή την τριάδα 456, αλλά όχι και τις δύο τριάδες μαζί;

86. Σάββατο 20 Ιουλίου 2013
http://www.mathematica.gr/forum/viewtopic.php?f=109&t=38543
Μία επιγραφή αποτελείται από ένα γράμμα του ελληνικού αλφαβήτου και ένα ψηφίο από το 0 έως και το 9. Πόσες τέτοιες επιγραφές υπάρχουν, αν ο αριθμός της θέσης του γράμματος πρέπει να είναι μεγαλύτερος ή ίσος με τον αριθμό της θέσης του ψηφίου; Για παράδειγμα δεν επιθυμούμε την επιγραφή Α3, επειδή ο αριθμός της θέσης του γράμματος Α είναι 1 και είναι μικρότερος από τον αριθμό της θέσης του ψηφίου 3 που είναι 4. Όμοια αποκλείονται οι επιγραφές Β5, Ε9 κλπ.

87. Τετάρτη 17 Ιουλίου 2013. 
http://www.mathematica.gr/forum/viewtopic.php?f=7&t=38455
Έχουμε 7 κυψέλες όπως στο συνημμένο σχήμα.
Μπορούμε να τοποθετήσουμε τους αριθμούς 1, 2, 3, 4, 5, 6 και 7 στις κυψέλες,
ώστε το άθροισμα κάθε γραμμής ή στήλης που αποτελείται από 3 κυψέλες να είναι το ίδιο;

88. Κυριακή 7 Ιουλίου 2013.
http://www.mathematica.gr/forum/viewtopic.php?f=109&t=38186
Η πρόσοψη μιας πολυκατοικίας έχει 12 παράθυρα. Θέλουμε να τα βάψουμε με τα χρώματα, Α, Β και Γ. Με πόσους τρόπους μπορεί να γίνει αυτό, αν κάθε χρώμα θα χρησιμοποιηθεί για τη βαφή τουλάχιστον ενός παραθύρου; Γενίκευση: Ο αριθμός των παραθύρων είναι k και ο αριθμός των διαφορετικών χρωμάτων n. 

89. 3 Δεκεμβρίου 2013. 
http://www.mathematica.gr/forum/viewtopic.php?f=22&t=41348
Τα τρίγωνα ABC και FED του συνημμένου σχήματος είναι ισόπλευρα, έχουν πλευρές παράλληλες ανά δύο και απέχουν την ίδια απόσταση. Αν οι πλευρές του FED έχουν ακέραιο μήκος, μπορεί να έχουν και οι πλευρές του ABC ακέραιο μήκος;

90. 1 Δεκεμβρίου 2013. 
Στο σχολικό εγχειρίδιο Γεωμετρίας για το Λύκειο από το οποίο διδάσκονται και αυτό το σχολικό έτος οι μαθητές, υπάρχει η εξής άσκηση εμπέδωσης στο 3ο Κεφάλαιο στη σελίδα 50. Δίνεται κύκλος (O, R). Αν N τυχαίο σημείο του κύκλου και M σημείο στην προέκταση του ON ώστε ON = MN, να βρεθεί ο γεωμετρικός των σημείων M, όταν το N διαγράφει τον κύκλο.
Γενικεύουμε την απλή αυτή άσκηση ως εξής: Δίνεται κύκλος (O, R)  και σημείο K του επιπέδου του. Αν N τυχαίο σημείο του κύκλου και M σημείο στην προέκταση του KN ώστε KN=NM, να βρεθεί ο γεωμετρικός των σημείων M, όταν το N διαγράφει τον κύκλο.
Τροποποίηση της προτεινόμενης άσκησης. Ο κύκλος (O, R) να αντικατασταθεί από το τετράγωνο ABCD. Το ερώτημα, όπως και οι συνθήκες της άσκησης να παραμείνουν οι ίδιες.

http://www.mathematica.gr/forum/viewtopic.php?f=111&t=41023
91.Σάββατο 16 Νοεμβρίου 2013. Το θέμα αυτό το βρήκα στο περιοδικό Mathematics Competitions Vol. 1, No 1, 1988, σελίδα 11. Οι μιγαδικοί z1 , z2 , z3 είναι διαφορετικοί, αλλά έχουν ίσα μέτρα.  Αν οι αριθμοί  z1 + z2z3, z2 + z1z3 , z3 + z1z2 είναι πραγματικοί, να αποδειχθεί ότι .z1z2z3=1

  http://www.mathematica.gr/forum/viewtopic.php?f=111&t=41021
92.Σάββατο 16 Νοεμβρίου 2013. Το θέμα αυτό είναι "πειραγμένο". Το είδα στο περιοδικό Mathematical Reflections. Οι αριθμοί  x, y, z ικανοποιούν τις εξής τρεις συνθήκες: x +y  + z =2, x^2 + y^2 + z^2 = 3, xyz = 4.
1. Να αποδειχθεί ότι τουλάχιστον ένας από τους αριθμούς αυτούς δεν είναι πραγματικός.
2. Να εξεταστεί αν μπορεί κάποιος από αυτούς τους αριθμούς να είναι πραγματικός.
3. Να υπολογιστεί η τιμή της παράστασης: 
 .(1/(xy+z-1) + 1/(xz + y-1) + 1/(yz + x -1)

93. Πέμπτη, 9 Ιανουαρίου 2014.

Κατασκευή τριγώνου εγγεγραμμένου σε κύκλο (μόνο για μαθητές) Δίνεται ένα τρίγωνο ABC και ένας κύκλος (O, R) . Να κατασκευαστεί ένα τρίγωνο όμοιο με το ABC εγγεγραμμένο στον κύκλο (O, R).

94. Δευτέρα 17 Φεβρουαρίου 2014. 
http://www.mathematica.gr/forum/viewtopic.php?f=112&t=42957

Δίνεται ο κύκλος (O, R) και μία ευθεία που διέρχεται από το O και τέμνει τον κύκλο στα σημεία B, P. Θεωρούμε τυχαίο σημείο A της διαμέτρου BP. Σχεδιάζουμε τα τετράγωνα  ABCD και AEHZ με το E να είναι σημείο του κύκλου (O, R), έτσι ώστε οι κορυφές των τετραγώνων να βρίσκονται προς το ίδιο ημιεπίπεδο σε σχέση με την ευθεία  BP όπως στο συνημμένο σχήμα. Ονομάζουμε L, M τα κέντρα των δύο τετραγώνων και N το μέσον του LM. Αν το σημείο A κινείται στο τμήμα BP, να βρεθεί ο γεωμετρικός τόπος του μέσου N.

http://www.mathematica.gr/forum/viewtopic.php?f=62&t=42839
95. Τετάρτη 12 Φεβρουαρίου 2014. 
Δίνονται δύο κάθετες ευθείες x, y  που τέμνονται στο σημείο O. Στην y υπάρχουν τα σταθερά σημεία A, B που ανήκουν στην ίδια ημιευθεία Oy και στην x το μεταβλητό σημείο C. Κατασκευάζουμε τα τετράγωνα ABCD και BCHZ, έτσι ώστε τα κέντρα τους G, I να βρίσκονται στη ίδιο τεταρτημόριο της γωνίας yOx, όπως στο συνημμένο σχήμα. Θέτουμε ένα ερώτημα και ζητάμε να το προσεγγίσουμε με όσους περισσότερους τρόπους μπορούμε.
ΕΡΩΤΗΜΑ: Καθώς το σημείο C κινείται στην ευθεία x, η απόσταση  GI μεταβάλλεται; Αν ναι, ποια είναι η μέγιστη και η ελάχιστη τιμή της;


http://www.mathematica.gr/forum/viewtopic.php?f=112&t=41674
96. Τετάρτη 18 Δεκεμβρίου 2013. Με αφορμή το πρόβλημα που τέθηκε στο Φόρουμ στη θέση  http://www.mathematica.gr/forum/viewtopic.php?f=110&t=41622 προτείνω μία δυσκολότερη εκδοχή του. 
Το τρίγωνο 
OAB είναι ορθογώνιο και ισοσκελές, ενώ το τρίγωνο ABC είναι ισόπλευρο. Μία ευθεία διέρχεται από την κορυφή C και τέμνει τις πλευρές  OA και OB στα σημεία P και Q αντίστοιχα, όπως στο συνημμένο σχήμα. Να βρεθεί η θέση της ευθείας, ώστε το γινόμενο OP.OQ να παρουσιάζει ελάχιστη τιμή.

http://www.mathematica.gr/forum/viewtopic.php?f=110&t=44319
97. Σάββατο 17 Μαίου 2014. Προτείνω ένα θέμα που παλιά το αποκαλούσαμε κατασκευή τριγώνου http://www.mathematica.gr/forum/viewtopic.php?f=112&t=8073.
Η εκφώνηση είναι προσαρμοσμένη στις εμπειρίες των σημερινών μαθητών.
Να βρεθούν οι κορυφές 
A, B, C του ισοσκελούς τριγώνου ABC με AC = BC, όταν γνωρίζουμε τη θέση των εξής σημείων.
1. Του σημείου M που είναι το μέσο του AC.
2. Του σημείου E που είναι η προβολή της κορυφής C στην AB.
3. Του σημείου D που είναι η προβολή της κορυφής A στην BC .

http://www.mathematica.gr/forum/viewtopic.php?f=69&t=43744
98. Κυριακή 6 Απριλίου 2014. Δίνεται τετράγωνοABCD και ένα σημείο P στο επίπεδό του. Να αποδειχθεί ότι τα μέσα των ευθύγραμμων τμημάτων PA, PB, PC, PD είναι κορυφές τετραγώνου. Πώς μπορεί να γενικευθεί η άσκηση αυτή;

http://www.mathematica.gr/forum/viewtopic.php?f=61&t=43741
99.  Κυριακή 6 Απριλίου 2014. Μπαίνει για απάντηση το εξής ερώτημα.
Υπάρχει συνεχής συνάρτηση f, η οποία να λαμβάνει οποιαδήποτε τιμή της, ακριβώς δύο φορές; Για παράδειγμα, η f(x) = x2 θα ήταν τέτοια, αν μπορούσε να λάβει την τιμή 0 για δύο ακριβώς διαφορετικές τιμές της μεταβλητής x.

http://www.mathematica.gr/forum/viewtopic.php?f=62&t=43736
100. Σάββατο 5 Απριλίου 2014. Αγαπητοί συνάδελφοι, δεν γνωρίζω αν αυτή η πρόταση είναι γνωστή. Θα ήμουν ευγνώμων αν κάποιος μου απαντήσει ότι την έχει συναντήσει κάπου και πού.
Δίδεται τετράγωνο ABCD και σημείο E στο εσωτερικό του. Κατασκευάζουμε τα τετράγωνα EAGF, EBJH, ECKL, EDOP όλα με την ίδια φορά όπως στο συνημμένο σχήμα. Να αποδειχθεί ότι τα κέντρα αυτών των τεσσάρων τετραγώνων είναι κορυφές τετραγώνου.
Ο φίλος Κώστας Δόρτσιος έδωσε μία απόδειξη, αλλά θα περιμένει τις δικές σας ιδέες.
Δύο σημαντικές παρατηρήσεις, ουσιαστικά επέκταση και γενίκευση της πρότασης.
1. Η πρόταση ισχύει και αν το σημείο 
E είναι εκτός ή και επί του αρχικού τετραγώνου.
2. Η πρόταση ισχύει και αντί για τετράγωνα κατασκευάσουμε ισόπλευρα τρίγωνα, κανονικά πεντάγωνα και γενικά κανονικά πολύγωνα του ίδιου είδους.

http://www.mathematica.gr/forum/viewtopic.php?f=111&t=45236
101. Κυριακή 13 Ιουλίου 2014. Να υπολογιστεί το πλήθος των πενταγώνων (κυρτών και μη κυρτών), των οποίων όλες τις πλευρές έχουν το ίδιο δεδομένο μήκος και το μέτρο των γωνιών τους σε μοίρες είναι ακέραιος αριθμός.

http://www.mathematica.gr/forum/viewtopic.php?f=110&t=45115
102. Σάββατο, 12 Ιουλίου 2014. 
Με αφορμή το πρόβλημα που έθεσε ο αγαπητός Demetres στη θέση  http://www.mathematica.gr/forum/viewtopic.php?f=95&t=44976., προτείνω μία γενίκευσή του.Για παράδειγμα, μπορεί ένα τυχαίο τρίγωνο να χωριστεί σε 2 δύο ισοσκελή τρίγωνα; Είναι γνωστό ότι αν είναι ισοσκελές με γωνίες 36, 72, 72 ή 108, 36, 36 αυτό πραγματοποιείται. Σε άλλα τρίγωνα μπορεί να συμβεί αυτό; Μετά να προχωρήσουμε στη μελέτη χωρισμού σε 3 ισοσκελή τρίγωνα κλπ. Η άσκηση αυτή είναι εμπνευσμένη από την άσκηση του KARKAR.
Να χωριστεί ένα τρίγωνο σε n πλήθος ισοσκελών τριγώνων, όπου n φυσικός > 1.

103. Παρασκευή 27 Ιουνίου 2014. 
http://www.mathematica.gr/forum/viewtopic.php?f=62&t=44984
Δίνεται κύκλος (K, R) και σημείο P εκτός αυτού με KP = d.  Μία ευθεία που διέρχεται από το P τέμνει τον (K, P) στα σημεία A και B. Έστω M το μέσον της χορδής AB.Η ευθεία KM τέμνει το έλασσον τόξο AB στο σημείο C.Ονομάζουμε O το μέσον του MC (του βέλους της χορδής AB). Να βρεθεί ο γεωμετρικός τόπος του σημείου συναρτήσει των R  και d, καθώς το σημείο A κινείται στον κύκλο, δηλαδή καθώς κινείται η ευθεία που διέρχεται από το σημείο P.

http://www.mathematica.gr/forum/viewtopic.php?f=109&t=44857
104.Παρασκευή 20 Ιουνίου 2014. 
Να λυθεί στους φυσικούς αριθμούς η εξίσωση:  2^x = 3^y + 5.

http://www.mathematica.gr/forum/viewtopic.php?f=110&t=45823
105. Παρασκευή 15 Αυγούστου 2014.
Καλή μέρα και χρόνια πολλά σε όσους εορτάζουν σήμερα.
Προτείνω να καταθέσουμε μία σειρά ασκήσεων Γεωμετρίας όπως εκείνες οι χρήσιμες συλλογές με τα τετράγωνα, τις γωνίες και άλλες θεματικές ενότητες που βοηθούν στην εξάσκηση σε γεωμετρικές έννοιες, αλλά ταυτόχρονα δείχνουν πώς κατασκευάζονται ασκήσεις και προβλήματα Γεωμετρίας. Φαίνεται να υπάρχει ένα ενδιαφέρον (έστω και τεχνητό, που επιβάλλεται από κίνητρα για εξετάσεις) για θέματα Γεωμετρίας και ταυτόχρονα φαίνεται να υπάρχει μία σχετική έλλειψη σε θέματα που αφορούν την λεγόμενη Γεωμετρία της κίνησης. Αν η πρότασή μου δεν θεωρηθεί χρήσιμη και ενδιαφέρουσα, απλά καταθέτω μία άσκηση.
Μία ευθεία a διέρχεται από σταθερό σημείο O. Δίνεται επίσης, ένα σταθερό σημείο P και ένα σημείο A της a για το οποίο ισχύει OA = d σταθερό μήκος. Η ευθεία a κινείται γύρω από το σημείο O σε ένα επίπεδο στο οποίο ανήκει και το σημείο P. Να βρεθεί ο γεωμετρικός τόπος των κύκλων που έχουν διάμετρο την PA.
Αν η ευθεία a κινείται στο χώρο γύρω από το σημείο O, ποιος είναι ο γεωμετρικός τόπος των κέντρων των κύκλων που έχουν διάμετρο την PA;

http://www.mathematica.gr/forum/viewtopic.php?f=110&t=45850
106. Κυριακή 17 Αυγούστου 2014

Δίνεται ένα επίπεδο (P) και δύο σταθερά σημεία του O και A.Μία ευθεία e του επιπέδου στρέφεται γύρω από το σημείο O.Να βρεθεί ο γεωμετρικός τόπος των κέντρων των κύκλων που διέρχονται από τα σημεία OA, και B, όταν η ευθεία e στρέφεται γύρω από το σημείο O.Ποιος είναι ο γεωμετρικός τόπος, όταν η ευθεία e στρέφεται στο χώρο γύρω από το σημείο O;
Στην ευθεία e ορίζουμε ένα σημείο B ώστε OB=d σταθερό μήκος.

http://www.mathematica.gr/forum/viewtopic.php?f=112&t=45873
107.Τρίτη 19-08-2014.
Δίνεται ο κύκλος (O, R) και ένα σταθερό σημείο του S.Ένα σημείο A του (O, R), είναι το κέντρο του κύκλου (A, r) με r σταθερό μήκος και r < R/2.Οι κύκλοι (O, R) και (A, r)τέμνονται στα σημεία B και C.Να βρεθεί ο γεωμετρικός τόπος των κέντρων των εγγεγραμμένων κύκλων στο τρίγωνο SBC, όταν το σημείο A κινείται στον κύκλο (O, R).

______________________________________________________________
Προβλήματα από το διάσημο
Ουγγρικό μαθηματικό περιοδικό Komal
των ετών 1997 και 1998.
September 1997
C. 473. Can it happen for a whole calendar year that no single Sunday falls on the seventh day of a month?
C. 474. A passanger has been walking for 3.5 hours, covering exactly 5 km in the course of any period of one hour. Is it possible that his average speed during his walk exceeded 5 km/h?
C. 475. In a triangle ABC, a point P on median CC1 is selected such that CP/PC1=m/n. Find the ratios in which P divides the segments of lines AP resp. BP lying inside the triangle.
C. 476. In a right circular cone, both the diameter of the base and the slant height are 20 cm. Find the maximum length of a 2 cm wide self-adhesive band that can be sticked on the lateral surface of the cone without any creasing, cutting, or overlapping.
Gy. 3142. Find the smallest positive integer that is divisible by 28, ends in 28 (in decimal system), and the sum of whose digits is 28.
Gy. 3143. Draw a chessboard on the plane. Let A1A2, ..., A32 and B1B2, ..., B32 denote the midpoints of the white and the black fields, respectively. Let furthermore P be any point of the chessboard. Prove that   A1P2A2P2A32P2B1P2B2P2B32P2.
Gy. 3144. Decide if 7/17 can be expressed as 1/a+1/b, where a and b are positive integers.

Gy. 3145. Solve inequality
Gy. 3146. Let a and b denote the lengths of the legs of a right triangle, and r its inradius. Prove that

Gy. 3147. Let P be a point inside a regular hexagon, and e a line incident to P and parallel to a side of the hexagon. Draw five additional lines passing through P such that among the six lines, any two consecutive lines form an angle of 30o. These lines divide the hexagon into 12 regions. Prove that these 12 regions can be divided into three groups such that the sum of the areas of the regions is the same in each group.
Gy. 3148. Let e1e1 and e1 be three pairwise skew edges of a cube. Select a point Ei on each edge ei for i=1,2,3. Find the locus of the centroids of the triangles E1E2E3.
Gy. 3149. In a triangle ABC, side AB is of unit length. The angles which include this side measure 15o and 60o, respectively. Find the lengths of the two other sides of the triangle without the help of trigonometric functions.
 F. 3187. Find those triangles which satisfy a2+b2+c2=8R2 where abc and R denote the lengths of the sides and the circumradius of the triangle, respectively.
F. 3188. There are given a circle centered at O, a point A selected on the circle, and a straight line d passing through O. A secant starting at A intersects the circle and line d at points B and D, respectively. Prove that upon rotating the secant about A, in any position, the circle incident to points OB and D passes through an other fixed point different from O.
N. 144. Let n denote an arbitrary positive integer. Prove that 2.(3n)! is divisible by n!(n+1)!(n+2)!.
N. 145. Does there exist a polynomial f(x,y,z) of real coefficients such that, f(x,y,z) is positive if and only if |x|, |y| and |z| are the sides of a triangle?
N. 147. Is it always true that in a tetrahedron, an inner point of each face can be selected such that they form the vertex set of a regular tetrahedron?

October 1997

C. 477. A game machine accepts two kinds of coins: red and green ones. For each coin the machine returns 5 coins of the other kind. One starts playing with 1 green coin. Can it happen that after a while he has as many green coins as red ones?
C. 478. In an arithmetic progression, the sum of the first n terms and the sum of the first 2n terms are A and B, respectively. Express the sum of the first 3n elements in terms of A and B.
C. 479. The lengths of the bases of a trapezium are a and c, respectively. Find the length of the segment parallel to the bases which halves the area of the trapezium.
C. 480. In a tetrahedron, two of the faces are equilateral triangles of unit side. The two other faces are right isosceles triangles. Find the volume of the tetrahedron.
Gy. 3150. In a certain town, each bus line has 3 stops. Any two bus-stops are connected with a line, and any two lines have a common bus-stop. How many bus lines may be there in the town?
Gy. 3155. Prove that in any triangle, a line passing through the incenter halves the perimeter of the triangle if and only if it halves the area of the triangle.
Gy. 3156. In a triangle, the segments of the medians lying inside the inscribed circle are all of the same length. Does this imply that the triangle is equilateral?
Gy. 3157. In a tetrahedron, the segments of the medians lying inside the inscribed sphere are all of the same length. Does this imply that the tetrahedron is regular?
F. 3192. Is it possible to move a knight on a 5x5 chessboard so that it returns to its original position after having visited each field of the board exactly once?
F. 3193. May a non-planar quadrilateral have only right angles?
F. 3194. In a right triangle ABC, construct the point D on the hypotenuse AB such that triangles DCA and DCB have equal inradii.
F. 3195. The whole surface of a cube shaped cake (including its bottom) is covered with chocolate. The cake is to be distributed among K people so that each of them receives the same amount of cake, and also the same amount of chocolate cover at the same time. In order to accomplish this task, we divide the cake into NxNxN alike cube shaped pieces, and give everybody the same number of pieces, taking care that the total surface area of the chocolate covered sides of the pieces one gets is also the same for everybody. Is it possible to realize this for an arbitrary K? At least how many pieces are necessary for K=1997?
N. 149. The sequence (an) is defined by the recursion a0=a1=1, (n+1)an+1=(2n+1)an+3nan-1. Prove that the sequence consists of integer numbers.
N. 150. There are given in the plane a parabola and points P and Q outside the parabola such that line PQ passes through the focus of the parabola. Draw two tangents to the parabola from each of points P and Q. Prove that the four points of intersection obtained this way lie on the same circle.
November 1997
C. 481. There is a company sitting at a round-table. They all leave the table for a while, and after returning to the table they all find that their neighbours differ from those they had previously. How many people may be sitting around the table?
C. 482. Prove that if y3x+1<x+y3 holds for some real numbers x and y, then they also satisfy inequality x3y+1<y+x3.
C. 483. The following question was presented at a Jeopardy show. Which plane figure has the property that its area is half the product of its diameters? According to te official answer, this is `the kite'. Are kites the only plane figures with the above property?
C. 484. In a right circular cone, the angle formed by the axis and a generator measures . Consider the ball inscribed in the cone. Find the ratio of the volume of the ball and that of the cone.
Gy. 3160. In an infinite arithmetic progression of distinct positive integers, we replace each term by the sum of its digits. Can it happen that the new sequence is again an arithmetic progression?
Gy. 3161. At a meeting there are n guest, including Mr. Smith. There is also a journalist there who is looking for Mr. Smith. He is aware of the fact that no one at the meeting knows Mr. Smith, who nevertheless knows everybody there. The journalist may approach any guest, point at someone, and ask the guest if s/he knows that specific person.
a) Can the journalist find Mr. Smith for sure with less than n questions?
b) What is the minimum number of questions he has to ask before he can identify Mr. Smith?
Gy. 3162. The lengths of the legs of a right triangle are a and b, respectively. Draw a circle of radius a centered at one endpoint of the hypotenuse, and also a circle of radius b centered at the other endpoint. Prove that the segment of the hypotenuse lying in the intersection of these two discs is as long as the diameter of the incircle of the triangle.
Gy. 3163. Find the minimum number of different lines determined by n non-collinear points in the plane.
Gy. 3164. In a triangle, let SMO and K denote respectively the centroid, the orthocentre, the incentre and the circumcentre of the triangle. Suppose that some two of these points coincide. In which of the six possible cases does this imply that the triangle is equilateral?
Gy. 3165. In a teterahedron it can happen that an altitude goes outside the polytope. How many such altitudes may a tetrahedron have?
F. 3196. Define a sequence an by a= 1, an+1 = sin an (= 0, 1, 2, ...). Prove that the sequence nan2 is bounded.
F. 3197. Some pieces are placed on an 8x8 chessboard such that there are exactly 4 pieces in each raw and coloumn of the board. Prove that there can be found 8 pieces among them such that no two of them are in the same row or coloumn.
F. 3200. We are given two segments AB and CD. Fix AB and move CD around, in a position parallel to AB, such that quadrilateral ABCD is a trapezoid in which a circle can be inscribed. Determine the locus of the midpoints of the segments CD.
F. 3201. Let M be an arbitrary interior point of a tetrahedron ABCD. Let NPQ and A1, respectively, denote the intersection points of planes BCMCDMBDM, and BCD with lines ADABAC and AM. Let finally D1B1 and C1 denote the points where lines A1NA1P and A1Q intersect the plane incident to A and parallel to BCD. Prove that A is the centroid of triangle B1C1D1.
ATTENTION! We correct and submit N. 146. again. (See KöMaL 1997/6.):
N. 152. There are given finitely many discs of unit radius in the plane such that the centres of any two of them are at least 10 units apart. Is it true that there exists a (not necessarily closed) polygon whose vertices are the centres of the given discs such that the polygon contains the centre of each disc, and each segment of the polygon intersects only those discs which are centered at its endpoints.

December 1997

C. 485. We are given a 100x100 array of numbers such that, for k=1,2,...,100, its kth row is an arithmetic progression whose first term is 1, and whose common difference is k. Find the largest entry along the diagonal that connects the lower left corner of the array to its upper right corner.
C. 486. How many different ways can some 1, 2, 5, 10 and 20 forint coins make 25 forints?
C. 488. The lateral surface of a frustum of a right circular cone is coloured with two colours; it is blue below the middle of its slant height and is red above that. Find the ratio between the radii of the bases of the frustum, given that the surface area of the blue part is twice as large as that of the red part.
Gy. 3166. Compare the following two numbers: 19971999 19991997
Gy. 3167. Prove that two positive integers a and b have the same parity if and only if there exist positive integers c and d such that a2+b2+c2+1=d2.
Gy. 3168. In a set of size 15, find 15 subsets, each of size 7, such that any two of them have exactly 3 elements in common.
Gy. 3169. Find all triangles with integer side-lengths in which one of the angles is twice as large as an other one.
Gy. 3170. A triangle ABC has a right angle at vertex C. The bisector of the angle at B intersects side AC and the circumcircle at points P and Q, respectively. Find the measures of the angles of the triangle, given that BP = 2PQ.
Gy. 3171. The lengths of the sides of a certain triangle are integers, one of them being 1. Is it possible that the inradius of the triangle is a rational number?
Gy. 3172. A right circular cylinder shaped vessel, whose axis is vertical, is filled up with water. The radius of the cylinder is 10 cm, its height is 25 cm. Find the amount of water which flows out of the vessel when it is tilted by 30o. Construct the angle by which the vessel should be tilted from its original position if we want to pour out half of the water it contains.
Gy. 3173. Given three straight lines in the space, each incident to a given point P, construct a plane through P which forms the same angle with each of the given lines.
F. 3203. Prove that any set of size 15 contains 15 subsets, each of size 6, such that any two of them have either 1 or 3 common elements.
F. 3204. There is given a heap of 923k pebbles. Two players play the following game. They move alternately, removing at each turn either 9, 2 or 3 pebbles from the heap. The one who cannot move, loses the game. Which player has a winning strategy?  
F. 3206. We are given a circle k1, and points H1H2 and P outside the circle. Construct a circle k2 passing through P such that, for i=1,2, the lengths of the tangents from Hi to k2 are the same as the lengths of the tangents from Hi to k1.
F. 3207. A regular tetrahedron, whose edges are 2 units long, is inscribed into a right circular cylinder in such a way that it has two vertices on the perimeter of each base of the cylinder. Find the volume of the cylinder.
N. 155. In an infinite sequence of positive integers, each term has the same number of divisors. Prove that there is an infinite subsequence in which any two terms have the same greatest common divisor.
N. 157. An nxn array consists of real numbers such that the entries are monotone decreasing in each row and each column of the array. Whenever we are given a number a, our task is to decide if the array contains that number or not. For that purpose, we may compare a with any entry in the array. How many comparisons do we have to make to solve the problem for sure, regardless of the specific value of a?
N. 158. There are given 9 convex subsets of (3-dimensional) space such that any 8 of them have a lattice point in common. Prove that there exists a lattice point which is contained in each set.

January 1998

C. 489. Joe and Charlie went on an excursion. At the end of their hike they arrived at a highway and decided to take a bus. Joe continued to walk forward to the next bus-stop at a speed of 4 km/h, while Charlie assumed that the previous stop was closer by, and thus headed in the opposite direction at a speed of 6 km/h. They each arrived just on time to catch the bus. Find out if Charlie's assumption was right, given that the bus travelled at a speed of 60 km/h.
C. 490. Prove that the difference between any two odd perfect squares is divisible by 8.
C. 491. Prove that every triangle has at most one such side which is smaller than the corresponding altitude.
C. 492. We measure the angles of inclination of a tower, emerging on level ground, at distances of 50 m and 100 m from its base, respectivley. The two measures sum up to 45o. Determine the height of the tower.
Gy. 3174. Ann and Bob play the following game. First they draw an nxn square and colour its horizontal sides with red and its vertical sides with blue. Ann starts the game and then they move alternately. At each move, Ann (Bob) chooses a small square which is not yet coloured and which already has at least one red (blue) side, and colours it with red (blue). The game ends when either Ann (Bob) connects the horizontal (vertical) sides with a chain of red (blue) fields such that any two consecutive ones share a common edge, in which case she (he) wins the game, or when a player, whose turn it is, cannot move, in which case the game is a draw. Determine those values of n for which one of the two players has a winning strategy.
Gy. 3177. There are two Hungarian teams, the Bolyai TC and the E\"otv\"os TK, among those qualified for the 16 best teams in the Europe Cup. How likely is that they are going to play against each other? (After each match, one team qualifies for the next round and the other one is eliminated from the contest.)
Gy. 3178. In a triangle of unit area, what is the smallest possible length of the second largest side?
Gy. 3179. The vertices of tetrahedron BRYG are coloured with blue, red, yellow and green, respectively. Next, the edges of the tetrahedron are coloured such that all the four colours are used and the colour of each edge agrees with the colour of one of its vertices. Prove that there can be found a vertex such that either the three edges starting at that vertex or the other three edges have the three colours blue, red and green, respectively.
Gy. 3181. In a right square based pyramid, the centres of the inscribed and circumscribed spheres coincide. Find the angle formed by any two neighbouring lateral edges.
F. 3208. Given that the positive numbers xyz satisfy x2+xy+y2=9, y2+yz+z2=16, zzx = 25, determine xy yz zx.
F. 3209. Consider a connected graph on n vertices and assign a real number to each edge of the graph which is to be called the value of that edge. For any path of the graph, the value of the path is defined as the largest value of any edge along the given path. For any two vertices x,y of the graph, let f(x,y) denote the smallest possible value of a path connecting x with y. Prove that the cardinality of the range of f is not greater than n-1.  
F. 3212. In a triangle ABCF denotes the midpoint of side BC and E denotes the common point of BC and the angle bisector starting at A. The circumcircle of triangle AEF intersects sides AB and AC at points B1 and C1, respectively. Show that BB1=CC1.
F. 3213. Consider two skew lines wich are extensions of two edges of a cube, respectively. A unit segment is placed arbitrarily along each line. In which position of the two segments has the tetrahedron determined by their endpoints a maximum volume?
N. 159. Prove that every convex n-gon can be dissected, with nn-3 pairwise non-crossing diagonals, into triangles such that no vertex of the polygon is inside the circumcircle of any triangle.
N. 161. The coefficients of a polynomial p are integers whose absolute values are not greater than 1998. Given that p(2000) is a prime number, prove that p cannot be written as a product of two polynomials, each of positive degree and of integer coefficients.
N. 162. The complete graph G on n vertices is to be decomposed as a union of complete bipartite graphs such that each edge of G belongs to exactly one of the bipartite graphs. Find the minimum number of bipartite graphs needed for such a decomposition.
February 1998
C. 493. Find all perfect squares with the following property: if we divide them by 11, the (partial) quotient is a prime number and the remainder is 4.
C. 494. Write the numbers 1,2,...,n in a row, in increasing order. In the next row, write the same numbers in decreasing order. Consider the difference of any two numbers, one of which is lying below the other. Determine the sum of the absolute values of all such differences.
C. 495. Prove that if the base angles of a trapezoid are not congruent, then the diagonal starting at the vertex of the smaller angle is longer than the other diagonal.
C. 496. The diagonals of a regular hexagonal prism have lengths 12 and 13. Calculate the volume of the polyhedron.
Gy. 3183. Some people, who know each other very well, are sitting at a round table. Some of them always tell the truth, but the others always lie. Each of them claims that he is veracious, but the kth person sitting on his right hand side is a liar. How many people can be there around the table?
Gy. 3184. In America, temperature is measured in Fahrenheit degrees. This is a linear scale in which the melting point of ice is taken as 32 degrees (32 oF) and the boiling point of water is taken as 212 oF. Someone gives us the temperature in Fahrenheit degrees, rounded to the nearest integer degree. We then convert it to the Celsius scale and round it to the nearest integer degree. Express, in centigrades, the maximum possible deviation of the temperature calculated this way from the actual temperature.
Gy. 3185. The numbers 1,2,...,n are assigned, in some order, to the vertices A1A2, ..., An of a regular n-gon.
a) Prove that the sum of the absolute values of the differences between adjacent numbers is at least 2n-2.
b) Find the number of arrangements in which the above sum is exactly 2n-2.
Gy. 3186. Construct a triangle, given the lengths of an altitude and a median, starting at the same vertex of the triangle, and the distance between that vertex and the orthocentre of the triangle.
Gy. 3187. Ann and Bob agreed to have a date somewhen between 5 and 5:30 p.m. Assuming that they arrive within the specified interval, find the probability that no one has to wait more than 10 minutes for the other.
Gy. 3188. Prove that every tetrahedron has a vertex such that the edges starting at that vertex can be rearranged to form a triangle.
Gy. 3189. Rotate a cube of unit edge by 60o about one of its diagonals. Calculate the volume of the intersection of the rotated cube with the original one.
F. 3214. Aladdin walked all over the equator in such a way that in each moment he either was moving to the west or was moving to the east or applied some magic trick to get to the opposite point of the Earth. We know that he travelled at most 19000 km's alltogether during his westward moves. Prove that there was a moment when the difference between the distances he had covered moving to the east and moving to the west, respectively, was at least half of the length of the equator.
F. 3217. Let s1s2s3 denote the lengths of the medians in a triangle, and let d1d2 and d3 denote the distances of a point P from the medians, respectively. Prove that one of the products s1d1s2d2 s3d3 equals the sum of the other two.
F. 3218. We are given a triangle A1A2A3 and a point P inside the triangle. Let Bi, for i=1,2,3, denote the foot of the perpendicular from P to line AiAi+1 (indices are taken modulo 3). In a similar manner, we can obtain a triangle C1C2C3 from triangle B1B2B3, and then triangle D1D2D3 from triangle C1C2C3. Prove that triangle D1D2D3 is similar to triangle A1A2A3.
F. 3219. Each face of a regular dodecahedron is coloured with one of four colours, red, blue, yellow and green, such that adjacent faces have different colours. Find the number of such edges for which one of the two faces incident to the edge is blue and the other one is green.
N. 164. Is there any number in the Fibonacci sequence whose six last digits (in decimal system) are all 9?
N. 166. We are given four points on a line in the following order: A,B,C,D. Moreover, we know that AB=CD. Is it possible to construct the midpoint of segment BC, if we are only allowed to use a straightedge? 
March 1998
C. 497. Is there any positive integer n such that 1.2.3.....(n-1).n, that is, n!, ends with exactly 100 zeros?
C. 498. Solve inequation x3+1>x2+x.
C. 499. There is given a circle of unit radius in the plane. A wreath of n congruent circles is formed around the given circle such that each circle in the wreath touches its two neighbours and also the given circle. Determine, in terms of n, the common radius of the circles. Calculate these radii, up to four digits precision after the decimal point, for the first four possible values of n.
C. 500. Is it possible to slip two fine arts albums, each 22 cm wide, and a 25 cm wide cookbook into 28 cm wide plastic bag? Assume that each book has a thickness of 1.5 cm.
Gy. 3190. We define a number 44...48...89 with the following procedure. First, we insert the number 48 between the two digits of 49. Next, 48 is inserted between the digits 4 and 8 of the resulting number. Finally, we repeat this second step a few times. Is it true that every number obtained this way is a perfect square?
Gy. 3191. Is it true that every integer has at least as many positive divisors of the form 4k+1 as that of the form 4k-1?
Gy. 3192. Find all pairs of positive integers A,B with the following property: if the decimal representation of A is placed in front of that of B, then the resulting number is a perfect square, equal to twice the product of A and B.
Gy. 3193. A basketful of peanuts is distributed into n piles. We gather the piles and rearrange them into n+k new piles (k>0). Prove that at least k+1 peanuts are transferred to smaller piles than the respective original piles that contained them. (H)
Gy. 3194. In a triangle, two of the altitudes are at least as long as the corresponding sides of the triangle. Find the angles of the triangle.
Gy. 3195. In a right triangle, parallel lines to each side are drawn through the incentre of the triangle. These lines divide each side of the triangle into three parts. Prove that the length of the middle part of the hypotenuse equals the sum of the lengths of the middle parts arising on the legs of the triangle.
Gy. 3196. There are given rays e0e1, ..., e6, each starting at the same point A of the plane. Each ray forms an angle of 30o with the one that precedes it in the given order. Let B0 denote the point on ray e0 such that A and B0 are a unit distance apart. Let B1 denote the orthogonal projection of B0 to e1. Similarly, denote by B2 the orthogonal projection of B1 to e2, and so on. Calculate the length of the polygonal path B0B1...B6 and the area of polygon B0B1...B6.
Gy. 3197. Four balls, each of radius r, are placed on a horizontal plane such that each of them touches two others and their centres form a square. A fifth ball of the same radius is placed on the top of these balls, touching all of them. Calculate the distance of the topmost point of the fifth ball from the given plane. (H)
F. 3220. A rectangular field is divided into 100 congruent rectangular plots with the help of lines parallel to the edges of the field. Neighbouring plots share a common edge. Initially, 9 plots of the field are overgrown with weeds. The weeds overgrow any plot whose (at least) two neighbours are already overgown. Is it possible that the whole field becomes overgrown sooner or later?
F. 3221. The numbers 1,2,...,n are assigned, in some order, to the vertices of a regular n-gon A1A2...An.
a) Determine the maximum possible value of the sum of the absolute values of the differences between numbers assigned to consecutive vertices.
b) Find the number of arrangements for which the above sum is maximum.
F. 3222. Given that n is a prime, prove that there is no positive integer x that satisfies equality (1n+2n+...+xn)+(1n+2n+...+(n-1)n)=1n+2n+...+(2n-1)n.
F. 3223. A polygon has exactly n axes of symmetry. How many sides may the polygon have?
F. 3224. In a certain convex polytope, every face has a central symmetry. Prove that at least six of the faces are parallelograms.
F. 3225. Find a set of points in 3-space which intersects every plane in at least one but not more than five points.
N. 167. There are given three unit discs in the plane. Let t1 and t2 denote the areas of the two regions in the plane which the discs cover exactly once resp. exactly twice. Prove that .
N. 170. Given that 15a+6b+4c+8d=0, prove that the equation ax3+bx2+cx+d=0 has a positive root.

April 1998

C. 501. One morning in the school the integers from 1 through a certain number were written on the blackboard. A student carefully erased one of them. There would have been nothing special about it if someone had not noted that the average of the remaining numbers had been . Try to figure out which number was erased.
C. 502. Let us denote the roots of equation x- 2bx b– c= 0 by x1 and x2. Prove that the roots of equation x2-2b(b2+3c2)x+(b2-c2)3=0 are and .
C. 503. Given are two isosceles triangles. The inscribed circles of the triangles touch the legs at their points of trisection closer to the base of the triangle in one case and further from the base of the triangle in the other case. In which case covers the inscribed circle a greater portion of the area of the triangle?
C. 504. The points ABCDEF lie in the 3 dimensional space. How are the points positioned if there exists a plane, in the same distance from each of the points, which separates points ABC from points DEF?
Gy. 3198. Do there exist positive integers abcd such that ab = cd and d is a prime number? 
Gy. 3199. Find all integer solutions to equation 1996+ 1998+ 1 = xy.
Gy. 3200. Let H be a 1000-element subset of the set {0, 1, 2, ..., 1998}. Prove that it has two not necessarily distinct elements a and b such that a+b is a power of 2.
Gy. 3201. A 9x9 chessboard is is packed with 1x2 dominoes such that only one corner of the board is left uncovered. Prove that the uncovered field of the board can be transferred to any other corner by shifting the dominoes about on the board.
Gy. 3202. A solid cube of edge length n cm is divided into smaller cubes each with egde length 1 cm. Determine those values of n for which a subset of the small cubes can be arranged to form a cube with edge length 2n cm. The latter cube may be hollow.
Gy. 3203. The lengths of the diagonals of a rhomboid are 2a and 2b, respectively. The sides of the rhomboid together with those tangents of the inscribed circle which are parallel to either diagonal form an octagon. Express the area of the octagon in terms of a and b.
Gy. 3204. The segments AB and CD are parallel. P is an interior point of segment BC, the lines AP and CD intersect in E. For which point P is the sum of the areas of triangles APB and CPE is minimum?
Gy. 3205. Semicircles are drawn externally on each side of the acute triangle ABC. The altitudes drawn from vertices A,B,C intersect the semicircles in points E,F,G, respectively. Prove that the hexagon AGBECF can be folded into a pyramid of base ABC.
F. 3226. In how many different ways can one colour the circles depicted on the figure such that each arm contains exactly two yellow and two blue circles? Two colourings are different if one cannot be obtained from the other using a rotation or a reflection.
F. 3227. Find all values of m for which polynomial yz- (x+y+z)m is divisible by (y+z)(z+x)(x+y). 
F. 3228. A bus line connects cities A and B. The buses leave in every 30 minutes. Each bus has a capacity of 50 passengers. Those who do not fit in the bus wait in the waiting hall, and those passengers who do not even fit there leave for B on foot. Between two consecutive buses either 0, 25, 50 or 75 passengers arrive, with probability 1/4, respectively. A new waiting hall is to be built to replace the old one. It is to be built from modules, each of capacity 25. At least how many such modules have to be used in order to ensure that the probability that one has to leave on foot is smaller than 1%?
F. 3229. In a triangle of side lengths a,b,c, the exradius belonging to side c is the geometric mean of the exradii belonging to sides a and b, respectively. Express c in terms of a and b.
F. 3230. Let S denote the centroid of triangle ABC. Erect perpendiculars from each vertex of the triangle to the external and internal bisectors of the angles belonging to the other two vertices. Prove that the sum of the squares of the lengths of the resulting perpendicular segments is 6(SA2+SB2+SC2).
F. 3231. Given are a sphere g and a point P in its interior. Let h1h2 és h3 be pairwise orthogonal chords passing through P. Each of the three planes determined by the pairs of chords intersect the sphere in a circle. Prove that the sum of the areas of the three circles is independent of the position of the chords.
N. 171. There are given lines e1e2, ..., en and a point P in the plane. Project P perpendicularly on the first line. Next, project its image on the second line, and so on. After projecting on the last line, project again on the first line, and continue the process. Prove that the different images obtained by the procedure form a bounded set.
N. 172. An automatic card shuffler is used to shuffle a deck of 2n cards. It can rearrange the deck according to the rule (1, 2, 3, ..., 2n-1, 2n) (2n, 1, 2n-1, 2, ..., n+1, n). Prove that the cards will be in their original order after at most 2n shuffles.  
N. 174. A tour of a tree graph is the following. A figure, moving along the edges of the graph, visits each vertex such that it travels along every edge exactly twice. We say that two figures tour the graph simultaneously if they start at the same time, not necessarily at the same vertex, and then they always move at the same time during their respective tours of the graph. They meet if they move to the same vertex or stay on the same edge of the graph at the same time. Let f(n) denote the largest integer for which there exist a tree on n vertices which f(n) figures can tour simultaneously such that no two of them meet. Prove that f(5k+1)=2k for every positive integer k.

May 1998

C. 505.  Let us call a calendar week even or odd according to the parity of the sum of the numbers of the days in the seven dates making up that week. How many even weeks can there be among any 52 consecutive weeks that counted from the first Monday in January?
C. 506. Find all pairs of integers m and n for which the equation
(2m-3)(n-1)x2+(2m-3)(n-1)(m-n-4)x-2(2m-3)(n-1)(m-n-2)-1=0 has integer solutions.
C. 507. Find the locus of the vertex of the parabola y=x2+tx+1 where t varies through the real numbers.
C. 508. Our train travels on a straight railway track at a constant velocity of 26 m/s. Looking out of the window, we notice a cylindrical grain silo. As we approach the silo for 5 seconds, our distance from the silo is reduced by 100 m, while the silo seems to turn through an angle of 5o. How much longer do we continue to get closer to the silo?
Gy. 3206. Given 2n distinct points on a line; points A1A2, ..., An are red and points B1B2, ..., Bn are blue. Consider all segments determined by the given points, and draw circles with these segments as diameters. A circle is then coloured red if the endpoints of its diameter are of the same colour. The other circles are coloured blue. Prove that the total length of the perimeters of the blue circles is at least as large as that of the red circles.
Gy. 3207. A certain number can be written in the form 3a2+32b2, where a and b are natural numbers. Multiply the number by 97. Prove that the new number can be expressed in that form too.
Gy. 3208. Solve the equation x(x-y) + y(y-z) + z(z-x) = 1 in the set of integers.
Gy. 3209. Which polynomials p(x) satisfy the following equality: (x-16)p(2x)=16(x-1)p(x)?
Gy. 3210. An inscribed trapezium is divided by its two diagonals into four triangles. Prove that each triangle has a circumcentre lying on one of the perpendicular lines dropped from the intersection point of the diagonals onto the sides of the trapezium.
Gy. 3211. There are given infinitely many rectangles in a Cartesian coordinate system such that each rectangle has an edge lying on the x axis and another one lying on the y axis. Moreover, the vertices of the rectangles opposite to the origin have integer coordinates. Prove that there are two rectangles such that one is contained in the other.
Gy. 3212. There are given 6 points in the plane, no three collinear. A few pairs of the given points are connected with line segments. At least how many segments must be drawn so that there should always be a triangle formed with its vertices being three of the given points?
Gy. 3213. Given a convex pentagon and a point P on its boundary, construct a line passing through P that halves the area of the pentagon.
F. 3234. In an all-aginst-all tournament, three teams are said to `beat around' each other if each of them wins exactly once when only games among those three teams are considered. At most how many beat-arounds may happen in a tournament with 23 participating teams?
F. 3235. Two 1998-gons are drawn in the plane such that the midpoints of their sides coincide. Does it imply that the polygons have equal areas?
F. 3236. Given a convex polygon K and a point P inside the polygon, prove that there exists a direction such that of all lines parallel to it, the one passing through P has the longest segment in K.
F. 3237. The diagonals of a certain four-sided prism pass through a common point. Prove that the prism is a paralleloepiped.
N. 175. There are n natural numbers written along a circle. We write the absolute value of their difference between every two consecutive numbers, and then erase the original numbers. We may repeat this procedure a finite number of times. Determine those values of n for which all numbers along the circle will be eventually zero, regardless of the particular choice of the original numbers.
N. 177. An automatic card shuffler is designed to shuffle, for any positive integer n, a deck of 2n cards according to the following rule: (1,2,3,...,2n) (2n,1,2n-1,...,n+1,n). We start with 1998 different cards. We complete the deck by adding 2n-1998 new cards to it, and feed the deck of 2n cards, in some order, to the machine. Is it possible to choose n and the order of the cards in such a way that every permutation of the original 1998 cards should occur in some finite number of shuffles?
______________________________

20  ΠΡΟΒΛΗΜΑΤΑ  ΓΕΩΜΕΤΡΙΑΣ
ΕΠΙΠΕΔΟΥ  ΜΑΘΗΜΑΤΙΚΩΝ  ΔΙΑΓΩΝΙΣΜΩΝ

Πολλαπλές προσεγγίσεις, σχόλια, υποδείξεις και γενικεύσεις

Ανδρέας Πούλος


       Τα παρακάτω προβλήματα προέρχονται από το βιβλίο των Ν. Βασίλιεφ και Α. Γιεγκόροφ «Πανενωσιακές Μαθηματικές Ολυμπιάδες της ΕΣΣΔ, 1961-1991», τόμος 1ος, στην ελληνική τους έκδοση από τον εκδοτικό οίκο Κάτοπτρο, Αθήνα, 1997. Τα προβλήματα αυτά επιλέχθηκαν με κριτήριο να μην είναι υπερβολικά δύσκολα, ενώ ταυτόχρονα να περιέχουν ευφυείς ιδέες και τεχνικές που να βοηθούν στην επίλυση και άλλων διαγωνιστικών προβλημάτων. Εκτός από τις λύσεις που προτείνονται από τους συγγραφείς, επιχειρούνται όσο είναι δυνατόν και άλλες προσεγγίσεις, συγκρίσεις και σχολιασμός σε σχέση με άλλα γεωμετρικά προβλήματα που έχουν τεθεί σε διαγωνισμούς. Θεωρήσαμε χρήσιμο να προστεθεί επιπλέον πληροφοριακό υλικό σχετικό με τα προβλήματα αυτά, όπως θεωρήματα, λήμματα, βιβλιογραφικές παραπομπές και αναφορές. Τα προ-βλήματα Γεωμετρίας και γενικά όλα τα θέματα που προτείνονται για μαθηματικούς διαγωνισμούς στην πρώην ΕΣΣΔ και στη σημερινή Ρωσία, διακρίνονται για ένα επίπεδο πρωτοτυπίας, απλότητας, κομψότητας και αντανακλούν μία υψηλού επιπέδου μαθηματική κουλτούρα και παράδοση.
Η σειρά που ακολουθούμε στην παρουσίαση και πραγμάτευση των προβλημάτων είναι η εξής:

  1. Εκφωνήσεις των προβλημάτων και υποδείξεις.
  2. Η βασική λύση και το αρχικό σχήμα.
  3. Άλλες λύσεις και προσεγγίσεις.
  4. Σχετικά θεωρήματα και παρεμφερή προβλήματα.
  5. Γενικεύσεις, νέα ζητούμενα και ερωτήματα.

        Το κάθε πρόβλημα πραγματεύεται και στις 5 ενότητες.
     Στην 1η ενότητα ως εκφώνηση, αφού ουσιαστικά πρόκειται για έναν κατάλογο προβλημάτων. Για ορισμένα από τα προβλήματα όμως δίνονται και υποδείξεις. Αυτές σχετίζονται με το πλήθος των περιπτώσεων που πρέπει να ελέγξει ο λύτης, με το είδος των πιθανών προσεγγίσεων, για παρά-δειγμα με τη χρήση Αναλυτικής Γεωμετρίας, διανυσμάτων, γεωμετρικών μετασχηματισμών κλπ. 
     Στην 2η ενότητα δίνεται η βασική λύση που προτείνεται από τους συγγραφείς με ελάχιστες τροποποιήσεις, που δεν σχετίζονται με την ουσία της λύσης, ακολουθούμενη από το αντίστοιχο σχήμα.
     Στην 3η ενότητα αποτυπώνεται μία προσωπική διδακτική παρέμβαση. Ο άμεσος στόχος είναι να δοθεί μία όσο γίνεται πληρέστερη πραγμάτευση των συγκεκριμένων προβλημάτων, τα οποία αποτελούν μία αφορμή για ενασχόληση σε βάθος με τα διαγωνιστικά προβλήματα. Ελπίζουμε ότι με τις υποδείξεις συναδέλφων, μαθητών και άλλων ενδιαφερόμενων σύντομα το κείμενο θα αυξηθεί ποσοτικά και θα  βελτιωθεί ποιοτικά.
     Στην 4η ενότητα φροντίσαμε τα θεωρήματα που αναφέρονται να είναι εντελώς σχετικά με τα προβλήματα που πραγματεύονται, ενώ με τον τίτλο «παρεμφερή προβλήματα» περιλαμβάνουμε υλικό από μαθηματικούς διαγωνισμούς και δημοσιεύσεις σε βιβλία και περιοδικά που σχετίζονται με αυτά τα 20 προβλήματα.
    Η 5η και τελευταία ενότητα θεωρούμε ότι είναι η πλέον αμφιλεγόμενη, επειδή πιθανώς να περιέχει γενικεύσεις πολύ πιο δύσκολες από τα συγκεκριμένα προβλήματα, γενικεύσεις που να απαιτούν νέα πιο σύνθετα εργαλεία, θεωρήματα και τεχνικές για να επιλυθούν. Φροντίσαμε ώστε να κρατηθούν ισορροπίες για να μην αποκλίνουμε από τους σκοπούς της εργασίας.
  
ΟΙ  ΕΚΦΩΝΗΣΕΙΣ  ΤΩΝ  ΠΡΟΒΛΗΜΑΤΩΝ

  1. Θεωρούμε τέσσερεις κύκλους με ακτίνες ρ1, ρ2, ρ3 και ρ4, τα κέντρα των οποίων είναι οι τέσσερεις κορυφές ενός ορθογωνίου και ισχύει ρ1 + ρ3 = ρ2 + ρ4  < d, όπου  d είναι η διαγώνιος του ορθογωνίου. Φέρουμε δύο ζεύγη εξωτερικών εφαπτομένων των κύκλων ρ1, ρ3 και ρ2, ρ4 αντίστοιχα. Να αποδείξετε ότι το τετράπλευρο που σχηματίστηκε από αυτές τις τέσσερεις ευθείες είναι περιγράψιμο σε κύκλο. (1η Πανρωσική Ολυμπιάδα, 1961).
  1. Στις προεκτάσεις των πλευρών ΑΒ, ΒΓ, ΓΔ και ΔΑ κυρτού τετραπλεύρου ΑΒΓΔ θεωρούμε τα σημεία Α΄, Β΄, Γ΄, Δ΄, για τα οποία ισχύουν: ΒΒ΄= ΑΒ, ΓΓ΄= ΒΓ, ΔΔ΄= ΓΔ΄ και ΑΑ΄= ΔΑ΄. Να αποδείξτε ότι το εμβαδόν του τετραπλεύρου Α΄Β΄Γ΄Δ΄ είναι πενταπλάσιο από το εμβαδόν του ΑΒΓΔ. (2η Πανρωσική Ολυμπιάδα, 1962).
  1. Από το μέσο Μ της βάσης ΑΓ ενός ισοσκελούς τριγώνου ΑΒΓ φέρουμε την κάθετο ΜΗ στην πλευρά ΒΓ. Το σημείο Ρ είναι το μέσο του τμήματος ΜΗ. Να αποδείξετε ότι το ΑΗ είναι κάθετο στο ΒΡ. (2η Πανρωσική Ολυμπιάδα, 1962).
  1. α) Κάθε μία από τις διαγώνιες ενός κυρτού τετραπλεύρου το διαιρεί σε δύο ισεμβαδικά σχήματα. Να αποδείξετε ότι το τετράπλευρο είναι παραλληλόγραμμο. β) Δίνεται κυρτό εξάγωνο ΑΒΓΔΕΖ. Αν κάθε μία από τις διαγώνιες ΑΔ, ΒΕ και ΓΖ το διαιρεί σε  δύο ισεμβαδικά σχήματα, να αποδείξετε ότι οι τρεις αυτές διαγώνιοι έχουν ένα κοινό σημείο. (3η Πανρωσική Ολυμπιάδα, 1963).
  1. Δίνεται ισόπλευρο τρίγωνο με πλευρά 1. Ένα τμήμα μήκους δ ολισθαίνει με τα άκρα του πάνω στις πλευρές του τριγώνου. Ποια είναι η ελάχιστη τιμή του δ, ώστε το τμήμα αυτό ολισθαίνοντας να σαρώνει ολόκληρο το τρίγωνο; (3η Πανρωσική Ολυμπιάδα ΕΣΣΔ, 1963).
  1. Σε ένα τρίγωνο ΑΒΓ φέρουμε τις διχοτόμους των γωνιών Α και Β. Κατόπιν φέρουμε από την κορυφή Γ ευθείες παράλληλες προς τις δύο διχοτόμους. Τα σημεία Δ και Ε είναι τα σημεία τομής των ευθειών αυτών με τις διχοτόμους. Αν ΔΕ // ΑΒ, τότε να αποδείξετε ότι το τρίγωνο ΑΒΓ είναι ισοσκελές. (3η Πανρωσική Ολυμπιάδα, 1963).
  1. Ποιο είναι το ελάχιστο πλήθος μη αλληλοκαλυπτόμενων τετραέδρων με τα οποία είναι δυνατόν να χωριστεί ένας κύβος; (4η Πανρωσική Ολυμπιάδα, 1964).
  1. α) Σε ένα οξυγώνιο τρίγωνο ΑΒΓ, το ύψος ΑΗ, που είναι το μεγαλύτερο από τα ύψη του τριγώνου, είναι ίσο με τη διάμεσο ΒΜ. Να αποδείξετε ότι η γωνία ΑΒΓ δεν είναι μεγαλύτερη από 60ο. β) Αν το ύψος ΑΗ ενός οξυγωνίου τριγώνου είναι ίσο με τη διάμεσο ΒΜ και με τη διχοτόμο ΓΔ, να αποδείξετε ότι το ΑΒΓ είναι ισόπλευρο. (1η Πανενωσιακή Ολυμπιάδα ΕΣΣΔ, 1967).
  1. Οι διάμεσοι χωρίζουν ένα τρίγωνο ΑΒΓ σε έξι τρίγωνα. Διαπιστώθηκε ότι οι τέσσερεις από τους εγγεγραμμένους κύκλους σε αυτά τα τρίγωνα είναι ίσοι. Αποδείξτε ότι το τρίγωνο ΑΒΓ είναι ισόπλευρο. (2η Πανενωσιακή Ολυμπιάδα ΕΣΣΔ, 1968).
  1. Ο εγγεγραμμένος κύκλος σε τρίγωνο ΑΒΓ εφάπτεται στην πλευρά ΑΓ στο σημείο Κ. Αποδείξτε ότι η ευθεία που συνδέει το μέσον της πλευράς ΑΓ με το κέντρο του εγγεγραμμένου κύκλου, διχοτομεί το τμήμα ΒΚ. (2η Πανενωσιακή Ολυμπιάδα ΕΣΣΔ, 1968).
  1. Δίνεται κύκλος, μία διάμετρός του ΑΒ και ένα σημείο Γ πάνω στη διάμετρο ΑΒ. Να κατασκευάσετε στον κύκλο δύο σημεία Χ και Υ, συμμετρικά ως προς τη διάμετρο ΑΒ, τέτοια ώστε η ευθεία ΥΓ να είναι κάθετη στην ευθεία ΧΑ. (4η Πανενωσιακή Ολυμπιάδα ΕΣΣΔ, 1970).
  1. Πόσες πλευρές μπορούν να έχουν μήκος ίσο με το μήκος της μέγιστης διαγωνίου ενός κυρτού πολυγώνου; (4η Πανενωσιακή Ολυμπιάδα ΕΣΣΔ, 1970).
  1. Έστω Ο το σημείο τομής των διαγωνίων ενός κυρτού τετραπλεύρου ΑΒΓΔ. Αποδείξτε ότι η ευθεία που διέρχεται από τα σημεία τομής των διαμέσων των τριγώνων ΑΟΒ και ΓΟΔ είναι κάθετη στην ευθεία, η οποία διέρχεται από τα σημεία τομής των υψών των τριγώνων ΒΟΓ και ΑΟΔ. (6η Πανενωσιακή Ολυμπιάδα ΕΣΣΔ, 1972).
  1. Το σημείο Ο, που βρίσκεται στο εσωτερικό ενός κυρτού πολυγώνου, σχηματίζει ισοσκελές τρίγωνο με οποιεσδήποτε δύο κορυφές του. Αποδείξτε ότι το σημείο αυτό ισαπέχει από τις κορυφές του πολυγώνου. (6η Πανενωσιακή Ολυμπιάδα ΕΣΣΔ, 1972).
  1. Δίνεται γωνία με κορυφή Ο και κύκλος που εφάπτεται των πλευρών της γωνίας στα σημεία Α και Β. Από το σημείο Α φέρουμε ημιευθεία παράλληλη προς την ΟΒ, η οποία τέμνει τον κύκλο στο σημείο Γ. Το ευθύγραμμο τμήμα ΟΓ τέμνει τον κύκλο στο σημείο Ε, ενώ οι ευθείες ΑΕ και ΟΒ τέμνονται στο σημείο Κ. Αποδείξτε ότι ΟΚ = ΚΒ. (7η Πανενωσιακή Ολυμπιάδα ΕΣΣΔ, 1973).
  1. Δίνεται ένα κυρτό ν-γωνο με παράλληλες κατά ζεύγη τις πλευρές του και ένα σημείο στο εσωτερικό του. Αποδείξτε ότι δεν είναι δυνατόν να φέρουμε από το σημείο αυτό περισσότερες από ν ευθείες, καθεμία από τις οποίες να διχοτομεί το εμβαδόν του ν-γώνου. (7η Πανενωσιακή Ολυμπιάδα ΕΣΣΔ, 1973).
  1. Δίνεται ένα τετράγωνο ΑΒΓΔ. Τα σημεία Ρ και Κ ανήκουν στις πλευρές ΑΒ και ΒΓ αντίστοιχα, έτσι ώστε ΒΡ = ΒΚ. Έστω Η το ίχνος της καθέτου, που άγεται από το σημείο Β προς την ΡΓ. Αποδείξετε ότι η γωνία ΔΗΚ είναι ορθή. (8η Πανενωσιακή Ολυμπιάδα ΕΣΣΔ, 1975).
  1. Τρεις μύγες κινούνται πάνω στις πλευρές ενός τριγώνου ΑΒΓ, έτσι ώστε το κέντρο βάρους του τριγώνου που σχηματίζουν να μένει στην ίδια θέση. Αποδείξτε ότι αυτό συμπίπτει με το κέντρο βάρους του τριγώνου ΑΒΓ. (Κέντρο βάρους είναι το σημείο τομής των διαμέσων του.) (9η Πανενωσιακή Ολυμπιάδα ΕΣΣΔ, 1976).
  1. Έστω δύο ισοσκελή ορθογώνια τρίγωνα. Οι κορυφές του μικρότερου τριγώνου βρίσκονται πάνω στις τρεις πλευρές του μεγαλύτερου τριγώνου (κάθε κορυφή σε διαφορετική πλευρά). Να υπολογίσετε την ελάχιστη τιμή του λόγου των εμβαδών αυτών των τριγώνων. (13η Πανενωσιακή Ολυμπιάδα ΕΣΣΔ, 1979).
  1. Κυρτό τετράπλευρο ΑΒΓΔ διαιρείται από τις διαγώνιές του σε τέσσερα τρίγωνα. Αν οι ακτίνες των τεσσάρων εγγεγραμμένων κύκλων στα τρίγωνα αυτά είναι ίσες μεταξύ τους, να αποδείξετε ότι το τετράπλευρο ΑΒΓΔ είναι ρόμβος. (13η Πανενωσιακή Ολυμπιάδα ΕΣΣΔ, 1979).
_________________________________________________________

Τα τεύχη του περιοδικού Quantum

Ο εκδοτικό οίκος Κάτοπτρο έκανε μία πολύ σημαντική προσφορά προς τους Έλληνες βιβλιόφιλους, για αυτούς που αγαπούν τη Φυσική, τα Μαθηματικά και γενικά τις Επιστήμες. Προσφέρει δωρεάν τα τεύχη της ελληνικής έκδοσης του περιοδικού Quamtum, τα οποία ο ενδιαφερόμενος μπορεί να τα διαβάσει ως αρχεία PDF στην παρακάτω ηλεκτρονική διεύθυνση:

Το είδα στην ανάρτηση του Λευτέρη Οικονόμου με την εξής αναφορά:

Με τη σύμφωνη γνώμη των εκδόσεων Κάτοπτρο, παρουσιάζουμε στην σελίδα αυτή σε μορφή αρχείων PDF τα 44 τεύχη του περιοδικού Quantum που εκδόθηκαν στα ελληνικά κατά την περίοδο 1994-2001. Το περιοδικό είναι ιδιαίτερα ενδιαφέρον μια που περιέχει εξαιρετικά άρθρα που άπτονται μια σειρά θεμάτων σύγχρονης Φυσικής και Μαθηματικών.

Τα τεύχη από το πρώτος έτος της έκδοσης του περιοδικού το 1994 έως το τελευταίο 2001 μεταφράζονταν και εμπλουτίζονταν για την ελληνική γλώσσα από ένα επιτελείο εκλεκτών επιστημόνων, πραγματικών διανοουμένων με απεριόριστη αγάπη για την Επιστήμη. Ο επιστημονικά υπεύθυνος του περιοδικού για την ελληνική γλώσσα ήταν ο νομικός Γιώργος Ευαγγελόπουλος, με πολύ γνώση καλή των Μαθηματικών, πολύ δυνατός λύτης διαγωνιστικών μαθηματικών προβλημάτων, ο οποίος είναι και σύμβουλος επί θεμάτων Επιστήμης του Προέδρου της Ελληνικής Δημοκρατίας. 
Την πρωτοβουλία για την δωρεάν ανάγνωση του περιοδικού Quantum είχε Λευτέρης Οικονόμου, σήμερα ομότιμος καθηγητής του Πανεπιστημίου Κρήτης και έως σήμερα ενεργός ερευνητής του Ιδρύματος Τεχνολογίας και Εφαρμογών. Στην επιστημονική επιτροπή της ελληνικής έκδοσης του περιοδικού περιλαμβάνονταν ο φίλος Μιχάλης Λάμπρου, καθηγητής στο Μαθηματικό Τμήμα του Πανεπιστημίου Κρήτης, ο Κώστα Σκανδάλης, καθηγητής Μαθηματικών στο ίδιο Τμήμα, και οι καθηγητές Φυσικής Στέφανος Τραχανάς, αργότερα υπεύθυνος του Εκδοτικού Οίκου των Πανεπιστημιακών Εκδόσεων Κρήτης, ο Θεοδόσης Χριστοδουλάκης κ.α.
Τα τεύχη του Quantum τα έχω όλα σε έντυπη μορφή από το πρώτο μέχρι το τελευταίο και σε ηλεκτρονική μορφή από τον φίλο Θεόδωρο Μπόλη. Μόλις πρόσφατα διαπίστωσα ότι είναι δωρεάν, και στο Διαδίκτυο, αν και αυτό είχε γίνει πριν καιρό. Θεώρησα λοιπόν, καλό να διαδοθεί αυτό το εξαιρετικό φυσικο-μαθηματικό περιοδικό σε ένα ευρύτερο κοινό με αντίστοιχα ενδιαφέροντα. 
Το περιοδικό είδα και στο ιστολόγιο για την Αστρονομία και την Αστροφωτογραφία στην παρακάτω θέση:

Να τονίσουμε ότι το περιοδικό στη ρώσικη έκδοση βασίστηκε σε ιδέα του Ακαδημαϊκού Φυσικού Πιότρ Καπίτσα τιμημένου με βραβείο Νόμπελ και υλοποιήθηκε από τον Ακαδημαϊκό, μαθηματικό Αντρέι Κολμογκόρωφ το 1964. Μπορείτε να βρείτε τη ρωσική έκδοση του περιοδικού στην θέση,
Το Quantum συντάσσεται έως σήμερα από διαπρεπείς επιστήμονες με μεγάλη εμπειρία στην "εκλαϊκευση" των Μαθηματικών και της Φυσικής. Στις χώρες της πρώην ΕΣΣΔ, το περιοδικό σε κάποιες χρονικές περιόδους έφθανε τις 200.000 τεύχη. Τα άρθρα του είναι προσαρμοσμένα στις γνώσεις των μαθητών Λυκείου, διακρίνονται για την πρωτοτυπία τους, είναι εξαιρετικά καλογραμμένα, ενθαρρύνουν την έρευνα, τη δημιουργική σκέψη, τη διεύρυνση των πλαισίων της φυσικομαθηματικής παιδείας. Ήταν ότι καλύτερο είχαμε ως φυσικομαθηματική κοινότητα για να προτείνουμε στον εαυτό μας, στους φίλους μας, στους μαθητές για αυτομόρφωση και γνήσια παιδεία. Δυστυχώς, η οικονομικές συνθήκες δεν επέτρεψαν την συνέχιση της μεταφρασμένης έκδοσης του περιοδικού. Τουλάχιστον, τα τεύχη που έχουν εκδοθεί και είναι αρκετά, να διαβαστούν και να διαδοθούν όπως αξίζει στο ποιοτικό τους περιεχόμενο.